Financial Analysis 2018 - 2019

Lakukan tugas rumah & ujian kamu dengan baik sekarang menggunakan Quizwiz!

"A company manager is making a decision concerning promotional strategy. What managerial accounting technique can assist in the implementation phase of her decision-making process? A. Budgeting expected revenue B. Recording and classifying transactions C. Comparing results to the budget D. Representing plans financially "

". B Recording and classifying transactions. Managerial accounting can aid in the implementation phase of a decision-making process through the practice of recording and classifying transactions. For example, if a payment is made to a designer, the payment will be recorded and classified for future reference. Budgeting expected revenue is a managerial accounting technique that would assist during the planning stages of the decision-making process. Comparing results to the budget would occur during the evaluation stage of the decision-making process. Representing plans financially is manifested in the budget and other reports that are completed during the planning phase, rather than during implementation. SOURCE: FI:660 SOURCE: Datar, S.M., & Rajan, M.V. (2014). Managerial accounting: Making decisions and motivating performance (p. 11). Upper Saddle River, NJ: Pearson Education."

"When preparing a statement of equity, what does a business deduct from its total net income to obtain its retained earnings? A. Dividends paid B. Depreciation of assets C. Sales tax payable D. Bad debt "

"A Dividends paid. Retained earnings are the income that a corporation keeps after paying dividends to its stockholders. This amount, which includes any forwarding balance of retained earnings from the previous accounting period, is reported on the company's statement of equity, which is also called the statement of retained earnings. The depreciation of assets, sales tax payable, and bad or uncollectible debt are not included on the statement of equity. SOURCE: FI:630 SOURCE: Peavler, R. (2017, March 17). What is a statement of retained earnings? Retrieved September 15, 2017, from http://bizfinance.about.com/od/yourfinancialposition/tp/Statement_Retained_Earnings.htm"

"Which of the following is an example of a finance professional exhibiting responsible behavior: A. During their first meeting, a financial planner asks Colin specific questions about his financial goals. B. Katherine wants to make a large commission, so she tries to sell a high-risk financial product to a client. C. A financial advisor delays contacting Olivia to advise her that the value of some of her stock is rapidly declining. D. During a dinner party, Ryan tells his friend specific details about a client's stock portfolio and financial status."

"A During their first meeting, a financial planner asks Colin specific questions about his financial goals. Responsible financial planners ask questions to determine their clients' financial goals and their comfort levels with risk. When financial planners understand their clients' financial needs and wants, they are better equipped to help their clients set realistic financial goals. The statement does not provide enough information to determine if Katherine's client is comfortable buying a high-risk financial product. If the client is not a risk-taker, then Katherine is acting irresponsibly. Sharing a client's confidential information with a friend and delaying bad news about an investment are irresponsible and unethical behaviors. SOURCE: FI:355 SOURCE: Peavler, R. (2018, April 26). Financial ethics will improve the profitability of your small business. Retrieved September 14, 2018, from http://bizfinance.about.com/od/fraud/a/Financial_Ethics.htm"

"Amanagerialaccountantcanhelpacompanymarketitsproductsby A. examining the relevant costs of advertising options. B. determining whether to make or buy a component. C. developing a target market analysis. D. creating a budget for hiring increases."

"A Examining the relevant costs of advertising options. Managerial accountants can use techniques such as relevant cost analysis to assist marketers in making decisions regarding advertising options. They can determine which will have the lowest and highest costs. Determining whether to buy or make a component is not necessarily a marketing decision. Developing a target market analysis would generally not be a task for a managerial accountant. Creating a budget for hiring increases is not directly related to marketing products. SOURCE: FI:660 SOURCE: Freedman, J. (2018, June 27). Why management accounting is important in decision- making. Retrieved September 14, 2018, from http://smallbusiness.chron.com/management- accounting-important-decisionmaking-53947.html"

"Stocks in which of the following industry sectors are least likely to be affected by changes in the business cycle: A. Healthcare B. Energy C. Technology D. Capital goods "

"A Healthcare. Healthcare, food, drink, and tobacco are generally considered to be non-cyclical industries, meaning that they are not greatly impacted by changes in the business cycle. The demand for such products and their stocks remains fairly steady regardless of the stage of the business cycle. Stocks in industry sectors such as energy, technology, and capital goods, on the other hand, often change in response to changes in the business cycle. The energy sector is affected by supply and demand of energy around the world, and political unrest can impact the industry considerably. Energy stocks seem to perform best late in the business cycle. Technology stocks are also affected by the business cycle, often in direct relation to the demand for capital goods. Technology stocks and capital goods stocks do well during business cycle expansion. SOURCE: FI:574 SOURCE: Moffatt, M. (2016, May 12). A beginner's guide to economic indicators. Retrieved September 18, 2017, from http://economics.about.com/cs/businesscycles/a/economic_ind.htm"

"When Penny started her job, she believed that lying was completely wrong. However, after two years, she has started to exaggerate financial projections and now believes that lies are only wrong when they cause direct harm. This is an example of A. incrementalism. B. self-interest. C. obedience to authority. D. conformity. "

"A Incrementalism. Incrementalism is the diminishing of ethical values over time. Penny's ethical standards have decreased over time. She might have once thought she would never exaggerate financial projections, but now she does it frequently and does not believe it is wrong. Self-interest is looking out for your own wellbeing and acting accordingly. Obedience to authority is the tendency to please authority figures and go along with them. Conformity is the tendency to behave in the same way that everyone else does, rather than exercising our own judgment. SOURCE: FI:355 SOURCE: LAP-FI-355—Money Morals (The Role of Ethics in Finance)"

" Which of the following is a true statement about stock information resources: A. It has never been easier to find stock information. B. There are very few stock information sources. C. Only wealthy investors have access to stock information. D. It is very difficult to find stock information. "

"A It has never been easier to find stock information. A few short years ago, only professionals and wealthy investors had access to in-depth stock information. Now, there is a great deal of easy-to-find information, all available to the individual investor. SOURCE: FI:274 SOURCE: Ryan, J.S. (2012). Personal financial literacy (2nd ed.) [pp. 377-380]. Mason, OH: South- Western Cengage Learning."

"When preparing a statement of changes in an owner's equity document, which of the following are specifically represented on the statement: A. New investments by the owner B. Property taxes paid by the owner C. Depreciation of company assets D. Lost revenue due to discharged debt"

"A New investments by the owner. New investments or withdrawals by the owner are specifically represented on a statement of changes in an owner's equity document, as well as the business's net income or loss. Property taxes, depreciation of company assets, and lost revenue are considered when determining net income or loss, but they are not specifically listed on the statement of changes in an owner's equity document. SOURCE: FI:630 SOURCE: Guerrieri, D.J., Haber, F.B., Hoyt, W.B., & Turner, R.E. (2012). Accounting: Real-world applications & connections (First-year course) [pp. 228-231]. Columbus, OH: Glencoe/McGraw-Hill."

"What is an advantage of setting ideal standards? A. Provide a lofty goal for employees to strive for B. Recognize that errors can occur in production C. Reduce the frustration associated with easily attainable goals D. Allocate time for re-work when errors occur"

"A Provide a lofty goal for employees to strive for. Some businesses challenge employees by setting ideal standards—standards that are or are almost unattainable because they're set so high. They identify the quantity and quality of output that can occur if no mistakes or errors occur and if workers do not need to rest. These businesses feel that these standards help employees to strive to achieve more than they would if attainable standards were set. Although some workers are challenged by the lofty goals, others question why they should bother with the work since the standards can't be reached. SOURCE: FI:662 SOURCE: Crosson, S.V., & Needles, B.E. (2014). Managerial accounting (10th ed.) [p. 218]. Mason, OH: South-Western Cengage Learning. "

"52 WEEK HI: 37.03 LOW:26.62 STOCK: Home Depot P/E:18.92 CLOSE:35.01 CHG: +.62 EARNINGS: 1.85 DIV:.95 YLD %: 2.7 Based on the information provided, which of the following statements about Home Depot is most likely to be accurate: A. The previous day's closing price was $34.39, $0.62 less than the last price for today. B. Home Depot's stock share price is 2.7 times higher than its earnings per share. C. The lowest price paid for Home Depot stock during the last year was $18.92. D. An investor who owned 100 shares of Home Depot's stock last quarter earned $27 in dividends."

"A The previous day's closing price was $34.39, $0.62 less than the last price for today. To calculate the previous day's closing price, investors can subtract the net change (the difference between the closing price and the previous day's closing price) from the last price for the day ($35.01 - $0.62 = $34.39). The price/earnings ratio (P/E) indicates how much higher the stock share price is than its earnings per share. So, Home Depot's stock share price is 18.92 times higher than its earnings per share. The lowest price paid for Home Depot stock during the last year was $26.62. An investor who owned 100 shares of Home Depot's stock last quarter earned $95 in dividends (100 shares X $0.95 per share = $95). SOURCE: FI:275 SOURCE: Ryan, J.S. (2012). Personal financial literacy (2nd ed.) [p. 394]. Mason, OH: South-Western Cengage Learning."

"Businesses usually invest in A. themselves. B. bonds. C. stocks. D. real estate. "

"A Themselves. Businesses invest money just as individuals do, with the hope for a healthy return. Often, businesses invest back into themselves by researching and developing new products, purchasing the latest and most efficient technologies, expanding into new markets, etc. They may also invest in stocks, bonds, or real estate, but putting money back into the company is the most common form of business investment. SOURCE: FI:077 SOURCE: LAP-FI-077—Invest for Success (Types of Investments)"

"What is one of the main purposes of a business's sales budget? A. To estimate income B. To forecast trends C. To predict the economy D. To track the competition "

"A To estimate income. The sales budget sets the financial pace for a business by estimating what amount of expenses and income can be expected from the sale of the business's goods and services. Sales budgets are extremely important because they estimate the major source of income, which helps a business determine if it will be profitable. Businesses take into consideration future trends, the state of the economy, and the level of competition when developing sales budgets. SOURCE: FI:106 SOURCE: LAP-FI-106—Money Tracks (Nature of Budgets)"

"What activity does accounting software help businesses do? A. Track credit transactions B. Place orders for raw materials C. Create and play AV media D. Create virtual situations "

"A Track credit transactions. Accounting software programs allow businesses to efficiently track various financial data and generate financial reports. Accounting software tracks bank and credit transactions, generates invoices, and maintains tax information. Some accounting software programs have inventory applications. Inventory software applications are used to track various types of stock, including office supplies, raw materials, and resale items. Multimedia software is used to create and play audio and video media. Simulation software applications are used to create virtual situations that are similar to real-life situations, such as aircraft-flight simulations. SOURCE: FI:352 SOURCE: Shanker, S. (n.d.). Information on accounting and information technology. Retrieved September 15, 2017, from http://www.ehow.com/about_6603681_information-accountinginformation-technology.html"

"What is an example of an internal stock information resource? A. The Value Line Investment Survey® B. A company's annual report C. Brokerage firm research D. A financial website portal "

"B A company's annual report. A company's annual report is an example of an internal source, because it comes from the company itself. The Value Line Investment Survey®, brokerage firm research, and financial web portals are all examples of external stock information resources. SOURCE: FI:274 SOURCE: Madura, J., Casey, M., & Roberts, S. (2014). Personal financial literacy (2nd ed.) [pp. 268- 278]. Upper Saddle, NJ: Pearson."

"Which of the following is most likely to have a negative effect on financial markets: A. Interest rates fluctuate daily. B. Business expansion slows quickly. C. Unemployment rates stabilize steadily. D. Consumer spending increases consistently. "

"B Business expansion slows quickly. Slower business expansion is one factor that indicates an economic recession. When business productivity slows, there is less work for laborers. When there is less work for laborers, businesses tend to lay off employees, causing unemployment rates to rise. Unemployment leads to less consumer spending because the displaced workers do not have income available to buy goods and services, including financial products. Fluctuating interest rates do not always indicate problems with financial markets because slight daily fluctuations are normal. Increased consumer spending usually indicates that the economy is rebounding, that people are working, and that people have more money to invest in the financial markets. SOURCE: FI:574 SOURCE: Moffatt, M. (2016, May 12). A beginner's guide to economic indicators. Retrieved September 18, 2017, from http://economics.about.com/cs/businesscycles/a/economic_ind.htm"

"Which of the following are examples of deposit-taking financial institutions: A. Commercial banks, corporations, and mutual savings banks B. Credit unions, savings and loan associations, and commercial banks C. Savings and loan associations, mutual savings banks, and insurance firms D. Commercial banks, credit unions, and brokerages "

"B Credit unions, savings and loan associations, and commercial banks. Deposit-taking financial institutions accept funds from customers (depositors) and pay interest on the funds. Credit unions, commercial banks, savings and loan associations (banks), and mutual savings banks are types of deposit-taking financial institutions. A corporation is a form of business ownership that is owned by stockholders who have purchased units or shares of the company. Brokerages are financial intermediaries that sell securities. Insurance firms sell insurance and securities. Corporations, brokerages, and insurance firms are not types of deposit-taking financial institutions. SOURCE: FI:336 SOURCE: Investopedia. (2015). Introduction—Types of financial institutions and their rates. Retrieved September 15, 2017, from http://www.investopedia.com/walkthrough/corporatefinance/1/financial-institutions.aspx "

"In the past 20 years, Country X has reformed its trade policies and currency exchange rate system. As a result, foreign investment is increasing, and the country's business activity and gross domestic product is rapidly growing. This is an example of a(n) A. integrated financial market. B. emerging market economy. C. recession-proof economy. D. industrial-based market. "

"B Emerging market economy. Emerging markets are growing markets (e.g., stock, bond, commodities, etc.) that result when a country initiates reforms that move it from a closed economy to an open or market economy. An integrated financial market involves a market with barrier-free financial flow. An economic recession is a six-month contraction in the gross domestic product (GDP). Because circumstances change and many factors affect the health of an economy, economic systems are not recession-proof. An industrial-based market is a market in which businesses sell goods and services to industrial businesses. SOURCE: FI:575 SOURCE: Investopedia. (2015). Emerging market economy. Retrieved September 15, 2017, from http://www.investopedia.com/terms/e/emergingmarketeconomy.asp "

"How does the finance function relate to company spending? A. It produces reports about spending. B. It plans and controls spending. C. It spends on investment only. D. It does not relate to spending."

"B It plans and controls spending. The finance function plans and controls spending throughout the company, making sure that it stays within the budget. It provides a central "hub" for monitoring company spending throughout all the different departments. The accounting function produces financial reports. Finance does not refer to spending on investments only. SOURCE: FI:354 SOURCE: LAP-FI-007—Money Matters (Role of Finance)"

"The primary purpose of managerial accounting is to provide accurate data about a business's activities that help internal users A. control external factors. B. make informed decisions. C. obtain necessary loans. D. understand corporate policies. "

"B Make informed decisions. Managerial or cost accounting involves obtaining and analyzing data about a business's activities (e.g., financial), which the business's employees use to make decisions. For example, managerial accounting information might reveal that sales are lower than anticipated and overhead is higher than expected for a specific time period. The appropriate employees can use the information to make decisions to help the business get back on track (e.g., cut unnecessary costs, increase efficiency). Businesses can monitor external factors, but cannot control them. Obtaining a loan might be a decision that a business makes as a result of the managerial accounting information it obtains. Not all businesses are legally established as corporations, so the primary purpose of managerial accounting is not to help employees understand corporate policies, which is usually a human resources function. SOURCE: FI:657 SOURCE: Investopedia. (2017). Managerial accounting. Retrieved September 27, 2017, from http://www.investopedia.com/terms/m/managerialaccounting.asp"

"Gathering and communicating financial information that businesses can use to plan the business's operations is the focus of __________ accounting. A. accrual B. managerial C. financial D. equity"

"B Managerial. Managerial accounting involves obtaining and analyzing financial data, which the business can use to make decisions about how to operate the business. For example, businesses gather financial information to develop budgets to guide the business's operations. They also analyze costs in relation to profit in order to make decisions about which products to produce and sell. Accrual is a method of accounting that records transactions at the time they occur even if no money changes hands at the time. The focus of financial accounting is preparing information for external users. Equity is the net worth of the business. SOURCE: FI:657 SOURCE: Investopedia. (n.d.). Managerial accounting. Retrieved September 14, 2018, from http://www.investopedia.com/terms/m/managerialaccounting.asp"

"The yen, peso, and euro are types of A. bonds. B. money. C. stock. D. dollars. "

"B Money. Money is the most common medium of exchange. Countries use different forms of currency. For example, the Japanese use the yen, Mexico uses the peso, many European countries use the euro, and the United States and Canada use the dollar. A bond is a lending security, and stock is an ownership security. SOURCE: FI:059 SOURCE: Phelps, C. (1999-2012). Types of foreign currency. Retrieved September 15, 2017, from http://www.ehow.com/about_5349743_types-foreign-currency.html"

"What type of variance analysis involves determining the difference between the standard costs and the actual costs of raw materials used for production? A. Selling price B. Purchase price C. Material yield D. Fixed overhead"

"B Purchase price. A variance is the difference between an established standard and the actual outcome. It is important for a business to monitor variances in the price of raw materials because they affect its bottom line. When a vendor increases the prices that it charges for raw materials, it costs the business (manufacturer) more money to produce an item. The business may need to find a vendor that charges less for the materials or parts, or it may need to increase its selling price—the amount that the business charges its customers for the finished good. Fixed overhead costs are expenses that do not change in relation to changes in sales volume and include things such as rent and employee salaries. Material yield analysis involves analyzing variances in the number of materials/products that are supposed to be used/sold with the actual number of materials/products that have been used/sold in a certain timeframe. SOURCE: FI:661 SOURCE: Bragg, S. (2017, August 30). Variance analysis. Retrieved September 14, 2018, from https://www.accountingtools.com/articles/what-is-variance-analysis.html"

"Which of the following is one of the main components of an income statement: A. Assets B. Revenue C. Taxes D. Liabilities"

"B Revenue. Revenue is the money received by resource owners and by producers for supplying goods and services to consumers. Revenue is an important component of an income statement because it indicates the total amount of money a business earns from all sources such as sales, interest, and dividends. A business's revenue should exceed its expenses in order for the business to make a profit. Taxes are calculated on the net income determined by the income statement. Assets are anything of value that a business owns. Liabilities are debts that a business owes. SOURCE: FI:094 SOURCE: LAP-FI-004—Watch Your Bottom Line (Income Statements)"

"A new employee left JRB Manufacturing two weeks after s/he completed the company's training program. The expense of training the new employee is a(n) __________ for the company. A. external cost B. sunk cost C. current asset D. capital liability"

"B Sunk cost. Sunk costs are costs that are incurred and cannot be recovered. In the situation described, the business spent money to train an employee who quit shortly thereafter. The business will not be able to recoup the training costs because the employee is no longer there to perform the work that s/he was trained to do. Training is an internal cost. Current assets include things such as inventory, cash, accounts receivable, etc. A capital liability is the responsibility incurred when purchasing a capital asset such as a building or a piece of equipment. This often involves borrowing a large amount of money to obtain the asset. SOURCE: FI:658 SOURCE: Bragg. S. (2017, August 23). Sunk cost. Retrieved September 14, 2018, from https://www.accountingtools.com/articles/what-is-a-sunk-cost.html"

"A company purchases an expensive new software system and divides the cost of it between the two departments that will use the system. This is an example of cost A. dividing. B. accumulation. C. allocation. D. precision. "

"C Allocation. Cost allocation is the assignment of costs to cost objects, such as a project, a department, a customer, etc. The company in this example allocates the cost of its new software system to two departments. This is not considered cost accumulation, dividing, or precision. SOURCE: FI:663 SOURCE: Averkamp, H. (2004-2015). What is cost allocation? Retrieved September 15, 2017, from http://www.accountingcoach.com/blog/what-is-cost-allocation"

"During its 20 years of operation, the Normandy Company has made very few changes to its accounting methods and system. As a result, the company's financial information is likely to be very A. reliable. B. comparable. C. consistent. D. conservative. "

"C Consistent. Consistent financial information results from the application of similar or identical accounting methods and systems over time within an organization. The Normandy Company has applied very similar accounting methods over its 20 years of operation. Therefore, the company's financial information is likely to be consistent. However, consistent information is not necessarily comparable, reliable, or conservative. There is no way of knowing if the company's accounting methods are similar to those of other businesses in the industry, so the accounting information may not be comparable. Reliable information is accurate and unbiased. The Normandy Company, though, could easily have inaccurate financial information, even though it is consistent. Conservative financial information does not overstate assets and sales or understate liabilities and expenses. There is no indication that the Normandy Company's information is conservative. SOURCE: FI:579 SOURCE: LAP-FI-009—By the Numbers (The Need for Financial Information)"

"Kenneth is easily able to guide the actions of his employees by referring to his budget. This is an example of which function of budgets? A. Controlling B. Planning C. Directing D. Monitoring"

"C Directing. Budgeting involves setting goals, executing those goals, and comparing results to those goals. Kenneth is executing his goals by directing his employees' actions with the help of his budget. Planning is setting goals. Controlling is comparing performance to expectations. Monitoring is tracking progress. They are all functions of budgets, but they are not specifically related to Kenneth's example. SOURCE: FI:662 SOURCE: Warren, C.S., Reeve, J.M., & Duchac, J. (2014). Managerial accounting (12th ed.) [p. 230]. Mason, OH: South-Western Cengage Learning. "

"Which of the following is an example of an indirect cost in an automobile factory: A. Shipping B. Glass C. Electricity D. Assembly line workers' salaries"

"C Electricity. Indirect costs are those which are not associated with one particular business activity and cannot easily be assigned to specific cost objects. Because electricity is used for many purposes in an automobile factory, it is considered an indirect cost. Glass is not an indirect cost because it goes toward specific products. Shipping costs, too, can be easily allocated to a particular product or business activity. Because the workers on an assembly line are contributing their efforts toward a particular product, their salaries can also be considered direct costs. Some employees' salaries are considered indirect, depending on the nature of their jobs. SOURCE: FI:663 SOURCE: Speilman, E. (2018, February 9). Direct costs vs. indirect costs: Understanding each. Retrieved September 14, 2018, from http://www.businessnewsdaily.com/5498-direct-costs- indirect-costs.html"

"Convergence and consolidation in the finance industry have resulted in a(n) A. more accurate picture of financial firms' financial condition. B. highly compartmentalized finance industry. C. one-stop shopping environment for consumers. D. single supervisory agency to regulate the finance industry."

"C One-stop shopping environment for consumers. Convergence, which is the merging of financial providers from different financial sectors, and consolidation, which is the merging of financial providers within the same institutional category, have created a one-stop shopping environment for consumers. Rather than having to go to different financial providers for different financial products and services, customers can find all of the financial products that they need—banking services, insurance policies, brokerage services, etc.—in one place, from one company. Rather than creating a highly compartmentalized finance industry, convergence and consolidation have created finance companies that want to be everything to everybody. Rather than specializing in any one service, many of today's financial firms offer a vast array of products and services. As financial firms merge and grow in size, it becomes more difficult for financial experts and the government to get an accurate picture of the firms' current financial condition. As finance corporations expand their product offerings to encompass several financial sectors, it becomes increasingly difficult to determine which agency should oversee the companies. There is no single supervisory agency to regulate the entire finance industry. SOURCE: FI:573 SOURCE: MBA Research and Curriculum Center. Introduction to finance course guide (pp. 4-49—4-51). Columbus, OH: Author."

"One disadvantage of marginal analysis is that it does not account for A. outsourcing. B. sunk costs. C. qualitative factors. D. labor costs. "

"C Qualitative factors. Marginal analysis only focuses on quantitative differences among choices and does not consider qualitative factors, such as customer service or trusted recommendations. Marginal analysis does account for sunk costs, which are costs that cannot be recovered and thus should not be included in marginal analysis. Outsourcing is using suppliers outside of an organization, and is one possible decision that could come out of a marginal analysis. Labor costs are generally considered when a marginal analysis is performed. SOURCE: FI:659 SOURCE: Crosson, S.V., & Needles, B.E. (2014). Managerial accounting (10th ed.) [pp. 332-344]. Mason, OH: South-Western Cengage Learning."

"Which of the following securities is most likely to be bought and sold on a money market: A. Short-term equity securities B. Long-term debt securities C. Short-term debt securities D. Long-term equity securities "

"C Short-term debt securities. Investors who put their funds into a money market are typically looking for a source of additional capital that will mature in a short period of time, usually less than one year. Examples of short-term debt securities include Treasury bills and certificates of deposit. Short-term debt securities often carry less risk than long-term debt and equity securities, which are bought and sold on capital markets. SOURCE: FI:337 SOURCE: Maverick, J.B. (2015, May 13). What's the difference between short-term investments and marketable securities? Retrieved September 15, 2017, from http://www.investopedia.com/ask/answers/051315/whats-difference-between-shortterminvestments-and-marketable-securities.asp"

"When reconciling your check register, it is important to __________ your balance. A. subtract your earned interest from B. add your debit card transactions to C. subtract all applicable fees from D. add ATM withdrawals to "

"C Subtract all applicable fees from. The check register is a central location to record and track all of the checks that you write and the amount of money you spend in your checking account. Each month, you receive a bank statement that details your checking account activity. You should reconcile your check register with your bank statement to ensure that they match. This step is important for many reasons. For example, if you forget to enter transactions in your check register, you may overdraft your account and be fined for insufficient funds. When reconciling your check register, you want to make sure that you have subtracted all fees including debit card transactions and ATM withdrawals. You should add the interest earned if your account is set up to earn interest. SOURCE: FI:069 SOURCE: Mycreditunion.gov. (2016). Understanding a check and balancing a checkbook. Retrieved September 15, 2017, from http://www.mycreditunion.gov/Pages/pocket-cents-understandinga-check-and-balancing-checkbook.aspx"

" Yazmin earned $5,000 from her summer job. If she invests it at a rate of 5%, how much will it be worth in two years? A. $5,025.50 B. $10,250 C. $5,500 D. $5,512.50 "

"D $5,512.50. The formula for calculating future value is Future Value = Present Value X (1 + Interest Rate)number of periods. To calculate the future value of Yazmin's investment, first add one to the interest rate (1+ 0.05 = 1.05). Then square that sum, which is the same as multiplying that sum by itself (1.052 = 1.05 X 1.05 = 1.1025). Finally, multiply the product by the original investment to determine the value of Yazmin's investment in two years (5,000 X 1.1025 = $5,512.50). SOURCE: FI:238 SOURCE: Carther, S. (2017, May 18). Understanding the time value of money. Retrieved September 13, 2018, from http://www.investopedia.com/articles/03/082703.asp "

"You want to have $8,000 in three years to purchase a car. You can earn 4% compound interest on your investment. How much should you invest today? A. $7,821.47 B. $7,364.10 C. $7,692.31 D. $7,111.75 "

"D $7,111.75. To determine how much you should invest today, calculate the present value of your investment using the formula Present Value = Future Value / (1 + Interest Rate)Number of Years. So, the present value of your investment is equal to $8,000 / (1 + 0.04)3 . To solve this equation, first add one to the interest rate (1 + 0.04 = 1.04). Next, raise this sum to the third power, which is the same as multiplying the sum by itself three times (1.043 = 1.04 X 1.04 X 1.04 = 1.1249). Finally, divide the future value of your investment by this product ($8,000 / 1.1249 = $7,111.75). To have $8,000 in three years, you need to invest $7,111.75 today. SOURCE: FI:238 SOURCE: Garrison, S. (n.d.). Time value of money: Self-paced overview. Retrieved September 15, 2017, from http://www.studyfinance.com/lessons/timevalue/index.mv"

"If Kayla wants to obtain the most current pricing for various stocks, she should A. review a business's annual report. B. pick up a business-oriented newspaper. C. log onto the "About Us" section of a business's website. D. access a financial website. "

"D Access a financial website. Pricing on the stock exchange fluctuates constantly, so accessing a financial website such as Bloomberg can give Kayla the most current information. A newspaper is typically printed on a daily basis so some of the information in the stock table is likely to be outdated by the time Kayla reads it. The ""About Us"" section of a business's website discusses the purpose of the business. The annual report is an overview of the business's performance for the previous year. Neither the ""About Us"" section of a business's website nor a business's annual report will contain the most current pricing for different stocks. SOURCE: FI:275 SOURCE: Madura, J., Casey, M., & Roberts, S. (2014). Personal financial literacy (2nd ed.) [pp. 268- 278]. Upper Saddle, NJ: Pearson."

"When should you record bank transactions in your check register? A. When checks bounce B. When bank statements are received C. As bank statements are reconciled D. As soon as each check is written "

"D As soon as each check is written. To keep accurate records, you need to record all information in the check register as each transaction occurs. Otherwise, the date, check number, or check amount may be recorded inaccurately. Waiting to record check information can result in difficulties with balancing bank statements when they arrive. This could also result in checks being returned to you for inadequate funds. SOURCE: FI:069 SOURCE: Mycreditunion.gov. (2018). Understanding a check and balancing a checkbook. Retrieved September 14, 2018, from http://www.mycreditunion.gov/Pages/pocket-cents-understanding- a-check-and-balancing-checkbook.aspx"

"What is the most negative information possible on a credit report? A. Collection agency report B. Late payment C. Lien D. Bankruptcy "

"D Bankruptcy. Bankruptcy is the last-resort action for poor credit. Late payments, liens, and collection agency reports all appear on a credit report, but none have as negative of an impact as bankruptcy. SOURCE: FI:568 SOURCE: Griffin, R. (2014, August 7). Bankruptcy may prevent qualifying for a credit card. Retrieved September 13, 2018, from http://www.experian.com/blogs/ask-experian/bankruptcy-credit- card/ "

"A business's sales may be strong, but that doesn't mean it's making as much money as it would like. In this situation, managers may analyze financial information to determine how to A. make purchases. B. create a budget. C. enter into contracts. D. boost profitability."

"D Boost profitability. When a business is not making as much money as it would like to, its managers look for ways to boost profitability. They do so by analyzing financial information. In this situation, managers would not be creating a budget, entering into a contract, or making purchases. SOURCE: FI:579 SOURCE: LAP-FI-009—By the Numbers (The Need for Financial Information)"

"The biggest risk to a real estate investment is A. inability to write checks. B. low rate of return. C. not being FDIC-insured. D. depreciation."

"D Depreciation. The biggest risk to a real estate investment is depreciation. If property values go down, the investor will not make a return and could even lose money. Only bank accounts are FDIC-insured, so not having that advantage does not necessarily make real estate a riskier investment than any other ownership investment. The same goes for the inability to write checks—this is not an advantage any other ownership investments have, either. As with most ownership investments, real estate has the potential for a high rate of return. SOURCE: FI:077 SOURCE: LAP-FI-077—Invest for Success (Types of Investments)"

"Certificates of deposit and collectibles are types of A. earnings. B. accounts. C. securities. D. investments."

"D Investments. There is a wide range of investments available to individuals who want the opportunity to increase their worth. Some investments are riskier than others, which means that individuals might lose money rather than make money. Certificates of deposit are safe investments because they pay a guaranteed amount of interest over a specific period of time. Collectibles are a riskier type of investment because the collectible, such as a baseball card, might lose value over time and not have any worth. Certificates of deposit and collectibles are not types of accounts or securities. Certificates of deposit earn interest while collectibles may or may not increase in value. SOURCE: FI:077 SOURCE: LAP-FI-077—Invest for Success (Types of Investments)"

"Which of the following is the most likely action that the government takes when a person misses the deadline for filing his/her income tax return: A. Allowance B. Refund C. Extension D. Penalty"

"D Penalty. A penalty is a punishment. The government depends on income tax to support public programs. When people do not provide income tax information when it is needed, the government often imposes a penalty, usually a fine or fee. Ongoing failure to report income can result in imprisonment. The government does not offer a refund or allowance for missing an important deadline. However, a person can file for an extension to avoid being penalized for missing a filing deadline. SOURCE: FI:074 SOURCE: IRS. (2014, April 18). Tips for taxpayers who missed the tax deadline. Retrieved September 10, 2018, from https://www.irs.gov/newsroom/tips-for-taxpayers-who-missed-the-tax-deadline"

"For budgeting purposes, which of the following would be considered a long-term financial goal: A. Buying a used car B. Saving for retirement C. Saving for an annual vacation D. Creating an emergency fund/"

. B Saving for retirement. Long-term goals, such as saving for retirement, take longer than a couple of years to achieve. Buying a used car, saving for an annual vacation, and creating an emergency fund should all take less than two years and are known as short-term financial goals. SOURCE: FI:064 SOURCE: Capital One. (2017). Savings basics - Setting short-term and long-term savings goals. Retrieved September 27, 2017, from http://www.capitalone.com/financial-education/savingsand-investing/savings/short-and-long-term-goals/

"To endorse a check, the payee must sign his/her name on the A. back of the check. B. signature block of the check. C. check's memo line. D. check's "Pay to the order of" section./"

A Back of the check. Endorsement involves signing the back of the check, which allows the funds to be transferred from the payer (check source) to the payee (check recipient). The payer completes the front of the check, which includes the "Signature" line and the "Pay to the order of" line, which identifies who can cash the check—usually the payee. The memo line is used to identify the purpose of the check. SOURCE: FI:560 SOURCE: Madura, J., Casey, M., & Roberts, S. (2014). Personal financial literacy (2nd ed.) [pp. 227- 229]. Upper Saddle, NJ: Pearson.

"If the cash from operating activities is consistently greater than the company's net income, the company's net earnings are of a(n) __________ quality. A. high B. low C. moderate D. excessive"

A High. The cash from operating activities is compared to the company's net income. If the cash from operating activities is consistently greater than the net income, the company's net income or earnings are said to be of a high quality. SOURCE: FI:541 SOURCE: AccountingCoach. (2004-2017). Cash flow statement (explanation). Retrieved September 13, 2017, from http://www.accountingcoach.com/cash-flow-statement/explanation

"Insurance is a key element in lifelong financial planning that A. provides protection against financial losses. B. protects a person from unethical salespeople. C. guarantees a solid financial future. D. is available only to property owners."

A Provides protection against financial losses. Insurance is designed to protect against financial losses, whether those losses are related to property, health, or even life. Insurance does not protect a person from unethical salespeople or guarantee a solid financial future. Insurance is available to property owners and non-owners alike. SOURCE: FI:064 SOURCE: Investopedia. (2018, June 28). Insurance. Retrieved September 20, 2018, from https://www.investopedia.com/terms/i/insurance.asp

"When setting financial goals, it is important to make sure that they are realistic and A. specific. B. general. C. conservative. D. durable./"

A Specific. A financial goal is a short- or long-term objective that is measured in terms of money. To accomplish financial goals, they must be specific, realistic (attainable), results-oriented, and time-bound. Financial goals are not general or durable, and they do not need to be conservative. SOURCE: FI:065 SOURCE: Madura, J., Casey, M., & Roberts, S. (2014). Personal financial literacy (2nd ed.) [pp. 41-45]. Upper Saddle, NJ: Pearson

"If individuals are not self-employed, which of the following is a type of earned income they might receive: A. Child support income B. Vacation pay C. Interest and dividends D. Trust disbursements"

B Vacation pay. If individuals receive paid vacation, they are being reimbursed for work done. Child support income is unearned because it is court ordered. Interest and dividends are unearned because an individual is not required to work for them. Trust disbursements also qualify as unearned income because no work is done to receive them. SOURCE: FI:061 SOURCE: TaxBraix.com. (2005-2016). What is the difference between earned, portfolio, and passive income? Retrieved September 13, 2017, from http://www.taxbraix.com/taxarticles/difference-between-earned-portfolio-passive-income.html

"What components must ""balance"" in a balance sheet? A. Assets plus liabilities should equal net profit. B. Income plus expenses should equal owner's equity. C. Assets minus liabilities should equal owner's equity. D. Income minus expenses should equal owners' equity./"

C Assets minus liabilities should equal owner's equity. The balance sheet gets its name from the fact that the figures it presents must balance each other. When the business subtracts what it owes (liabilities) from what it owns (assets), the result should be the same as the owner's equity. This is the total net worth of the business. Income, expenses, and net profit are all found on an income statement. SOURCE: FI:085 SOURCE: LAP-FI-085—Show Me the Money (Nature of Accounting)

"How do businesses transfer risk? A. Screen employees B. Implement security measures C. Check for hazards D. Obtain insurance"

D Obtain insurance. Risk is the possibility of loss. There are many ways to reduce the possibility of loss. One way to reduce the possibility of loss is by transferring the risk, which is done by obtaining insurance. Insurance is a contractual agreement in which one company (insurer) will pay for specified losses incurred by the other company (insured) in return for installment payments (premium). Implementing security measures, checking for hazards, and screening employees are ways to control and prevent certain types of risks. SOURCE: FI:081 SOURCE: Farese, L.S., Kimbrell, G., & Woloszyk, C.A. (2012). Marketing essentials (pp. 807-808). Columbus, OH: Glencoe/McGraw-Hill.

"One way money functions in society is that it helps people communicate a product's A. attributes. B. income. C. market. D. value./"

D Value. Money is the most common medium of exchange in society. People exchange money for goods and services that they want. The price, or the amount of money a person is willing to pay for the product, communicates the product's value or worth. Income is money resource owners receive for supplying goods and services. Market is a customer or potential customer who has an unfulfilled desire and is financially able and willing to satisfy that desire. Attributes are a product's characteristics. SOURCE: FI:060 SOURCE: Farese, L.S., Kimbrell, G., & Woloszyk, C.A. (2012). Marketing essentials (pp. 365-369). Columbus, OH: Glencoe/McGraw-Hill.

"Which of the following is a risk a business might face if it obtains a long-term loan from a financial institution and pays off the loan before it is due: A. Being charged a penalty B. Losing interest on the account C. Paying a processing fee D. Receiving a variable rate" "

" "A Being charged a penalty. When businesses obtain long-term loans, they agree to pay a certain amount of interest to the lending institution over a period of years. If a business pays off the loan before it is due, the financial institution does not get the interest it expected to receive. In such cases, the financial institution may charge the business a penalty for prepaying the loan. This penalty may be sizeable depending on the amount of the loan. A financial institution, rather than a business, loses interest on the account. A business might negotiate a variable rate for a long-term loan and pay a processing fee to originate the loan. SOURCE: FI:041 SOURCE: Business Filings. (2012, May 24). The real cost of borrowing money. Retrieved September 17, 2018, from https://www.bizfilings.com/toolkit/research-topics/finance/business- finance/the-real-cost-of-borrowing-money"

"Managerial accounting differs from financial accounting because managerial accounting A. helps people within a business rather than outside of the organization. B. does not focus on planning and budgeting. C. is governed by federal regulation. D. has less impact on decision-making." "

" "A Helps people within a business rather than outside of the organization. Managerial accounting is focused on providing financial information for internal organizational use. In contrast, financial accounting information is used outside of the organization. Managerial accounting does focus on planning and budgeting. Financial accounting, rather than managerial accounting, is governed by federal regulation. Finally, managerial accounting has more impact on decision-making than financial accounting because it is used within the organization as decisions are made. SOURCE: FI:657 SOURCE: Jan, I. (2011-2013). Managerial accounting. Retrieved September 19, 2017, from http://accountingexplained.com/managerial/introduction/"

"One reason why banks might be reluctant to lend money to a start-up business is because the business A. has a solid business plan. B. will not have collateral. C. has no record of repaying loans. D. must incorporate first." "

" "C Has no record of repaying loans. Banks look at many factors to determine if a business is worthy of credit. Start-up businesses borrow money based on the entrepreneur's past record of loan repayment because the business is new and has no previous debt repayment record. If a business incorporates, other avenues of securing capital are available. If start-up businesses have sufficient collateral and a solid business plan, it is easier for them to obtain financing from a bank. SOURCE: FI:034 SOURCE: Carabelli, C. (2012, April 9). What do banks look at for loans? Retrieved September 19, 2017, from http://smallbusiness.chron.com/banks-look-loans-42003.html"

"By understanding the time value of money concept, you know that A. saving money can buy time on debt payment. B. it is better to receive $101 a year from now than $100 today. C. it is better to receive $100 today than $101 a year from now. D. time and money are related by due date. "

"C It is better to receive $100 today than $101 a year from now. According to the concept of the time value of money, the sooner money is received the more time you have to let it grow. Receiving money later costs you the opportunity to earn interest or other appreciation. Saving money will not buy time on debt payment. Time and money are not related by due date. SOURCE: FI:062 SOURCE: Carther, S. (2018, May 18). Understanding the time value of money. Retrieved September 14, 2018, from http://www.investopedia.com/articles/03/082703.asp"

"To be useful to businesses, financial information should be A. vague and transferable. B. reliable and creative. C. timely and understandable. D. sensitive and logical."

"C Timely and understandable. Businesses obtain internal and external financial information to make decisions about their business activities. To make sound decisions, a business needs reliable and timely information that is presented in an understandable or logical way. The information should also be specific and relate to the situation at hand. Financial information does not need to be creative, transferable, or sensitive. SOURCE: FI:579 SOURCE: LAP-FI-009—By the Numbers (The Need for Financial Information)"

"Chris is at the grocery store deciding between two very similar candy bars. One is $1.50, and the other is $3.00. He decides to buy the $1.50 candy bar because it is cheaper. Chris is using money as a ____________ to make his decision. A. store of value B. medium of exchange C. unit of measure D. commodity"

. C Unit of measure. Money can serve as a unit of measure to determine or compare the relative worth of different products. Chris compares the monetary value of the two candy bars and selects the less expensive one. While money does serve as the medium of exchange in this example, the fact that Chris is using money instead of some other medium of exchange does not directly impact his decision. Money would serve as the medium of exchange even if he purchases the more expensive candy bar. Store of value means that money can be held over time and still have purchasing power at a later date. The money that Chris is using is not a commodity because it does not have another use. SOURCE: FI:060 SOURCE: CliffsNotes. (2016). Functions of money. Retrieved September 20, 2018, from https://www.cliffsnotes.com/study-guides/economics/money-and-banking/functions-of-money

"Financial information is not useful if it does not contain understandable A. language. B. footnotes. C. investment analysis. D. recommendations./"

A Language. Financial information is not useful if it does not contain understandable formatting and language. It is often not necessary for readers to understand each footnote in a financial report. People use financial information to analyze potential investments, but the information itself does not have to come in the form of an analysis. It does not have to include recommendations to be understandable. SOURCE: FI:579 SOURCE: LAP-FI-009—By the Numbers (The Need for Financial Information)

"Emma is reviewing the sales budget in relation to the actual sales generated for a set time frame. How is Emma using financial information in this situation? A. To compare estimated performance with actual performance B. To establish new quality standards and benchmarks C. To identify external economic trends D. To analyze the impact of aging accounts"

A To compare estimated performance with actual performance. Businesses rely on accurate financial information to make sound business decisions. In the situation provided, Emma is comparing one financial report (budget) with another financial report (actual sales) to determine if the business performed as expected—if it achieved its sales goals. Although external economic conditions can affect sales, Emma is analyzing internal financial information rather than identifying external economic trends. Emma is not establishing new quality standards and benchmarks or analyzing the impact of aging accounts. SOURCE: FI:579 SOURCE: LAP-FI-009—By the Numbers (The Need for Financial Information)

"Which of the following are listed as long-term assets on a balance sheet: A. Capital equipment, mortgage, and inventory B. Land, cash, and mortgage C. Vehicles, insurance, and accounts payable D. Buildings, vehicles, and capital equipment"

D Buildings, vehicles, and capital equipment. A balance sheet is a financial statement that captures the financial condition of the business at that particular moment. The balance sheet shows a business's assets, liabilities, and owner's equity. Long-term assets are items of value—buildings, vehicles, and capital equipment—that businesses use to generate revenue for more than a year. Inventory and cash are listed as current or short-term assets, which involve levels that tend to fluctuate. The mortgage is a long-term liability, and accounts payable (e.g., insurance premiums) are current liabilities. SOURCE: FI:093 SOURCE: LAP-FI-010—The Right Balance (The Nature of Balance Sheets)

"When determining financing needs, what factor should businesses consider that will help decide whether they can repay the debt? A. Liquidity B. Depreciation C. Equity D. Inventory" "

" "A Liquidity. Liquidity refers to a business's current assets, such as income or savings, that can be used to pay expenses. Assets that are in the form of cash or tangibles that can be sold easily to generate cash add to a company's liquidity. When determining financing needs, businesses usually consider their liquidity to decide if they have sufficient funds available to repay the debt. For example, if a business was barely covering expenses and had no liquid assets, it probably would decide not to borrow money because it would be difficult to make payments on the loan. Depreciation is a reduction in value of goods occurring over a period of time. Equity is the value of the business after all expenses and debts have been subtracted. Inventory is all the stock a business has on hand. SOURCE: FI:043 SOURCE: Mueller, J. (2018, May 8). Understanding financial liquidity. Retrieved September 17, 2018, from https://www.investopedia.com/articles/basics/07/liquidity.asp"

"When a business has been running successfully for several years, what would be a probable next financial step? A. Purchasing a second location B. Acquiring start-up funds C. Deciding on financial goals D. Determining product prices" "

" "A Purchasing a second location. If a business has been running successfully for several years, it may be in the expansion stage of its lifecycle. This could mean that the company would consider purchasing a second location. The company would acquire start-up funds, decide on financial goals, and determine product prices at a much earlier stage in development. SOURCE: FI:339 SOURCE: Ryan, K. (2015, March 23). The 4 stages of the business lifecycle and how to recognize what you need next. Retrieved September 19, 2017, from http://www.roseryan.com/blog/2015/03/the-4-stages-of-the-business-lifecycle/"

"What sales forecasting technique creates accurate forecasts for individual products? A. Sales force composite B. Delphi C. Survey of buyer intentions D. Jury of executive opinion" "

" "A Sales force composite. There are several advantages to the sales force composite. More accurate forecasts for individual products can usually be obtained because sales personnel have first-hand knowledge about demand for certain products as a result of their constant contact with customers. It is an inexpensive method to use because it has few costs other than the time required. It also provides as much detail as the business might need because the business can forecast sales for individual products and for an overall company sales forecast. The Delphi sales-forecasting method can take a lot of time; therefore, it is expensive. The survey of buyer intentions sales forecasting method is not always accurate since customers sometimes can't make accurate predictions of their buying intentions. The jury of executive opinion sales forecasting method may be too optimistic or too conservative depending upon the executives' viewpoints. Therefore, some forecasters consider this a form of guesswork, thereby requiring the use of other forecasting methods to support it. SOURCE: FI:096 SOURCE: Insight Central. (2011, February 10). Forecast Friday topic: Expert judgment. Retrieved September 19, 2017, from https://analysights.wordpress.com/tag/sales-force-composite- forecasts/"

"Which of the following payback periods in a cost/benefit analysis would businesses prefer if all other things were equal: A. 2 years B. 1 year C. 3 years D. 4 years" "

" "B 1 year. Businesses prefer a short payback period. This is the point at which a decision begins to pay for itself. In other words, the payback period is the time it takes for the benefits of a change to repay its costs. Another term for payback period is break-even point. SOURCE: FI:357 SOURCE: LAP-FI-011—Running the Numbers (Cost-Benefit Analysis)"

"Businesses often obtain cash to produce and sell goods by borrowing against their A. current liabilities. B. accounts receivable. C. operating expenses. D. promissory notes." "

" "B Accounts receivable. Accounts receivable are all the monies owed to a business by its customers. Businesses in need of cash to continue producing and selling goods often use their accounts receivable as collateral when borrowing from financial institutions. In effect, the businesses are turning over the proceeds from their receivables to the lenders in exchange for immediate cash to continue operating. A promissory note is a legal form signed by a borrower promising to repay a loan. Liabilities are debts that the business owes. Operating expenses are all of the expenditures necessary to keep the business running. SOURCE: FI:023 SOURCE: Investopedia. (2018, May 29). Accounts receivable financing. Retrieved September 17, 2018, from https://www.investopedia.com/terms/a/accountsreceivablefinancing.asp"

"Which of the following categories of information are found on a balance sheet: A. Income, expenditures, profit B. Assets, liabilities, owner's equity C. Assets, liabilities, margin D. Revenues, expenses, profit" "

" "B Assets, liabilities, owner's equity. A balance sheet is a financial statement that captures the financial condition of the business at that particular moment. The statement first lists assets, anything of value that a business owns; then liabilities, or debts, that the business owes; and finally owner's equity, or the amount the owner has invested in the business, plus or minus profits and losses. Revenues, or income; expenses, or expenditures; and profit are components of an income statement, or profit-and-loss statement. Margin is a term that describes the difference between net sales and cost of goods sold. SOURCE: FI:085 SOURCE: LAP-FI-085—Show Me the Money (Nature of Accounting)"

"In managerial accounting, the number of hours that a business runs its machinery during the production process is classified as a(n) A. production output. B. cost driver. C. unexpected expense. D. inflexible transaction." "

" "B Cost driver. A factor that causes a change in the cost of an activity is called a cost driver. Changes such as the amount of labor used, the number of machines used, the power used to run the machinery, and the number of hours that the machinery is operated are cost drivers. Increases in these activities increase the costs of making outputs (products), while decreases in these activities decrease the costs of making the outputs. Cost drivers are not unexpected expenses or inflexible transactions. SOURCE: FI:657 SOURCE: Investopedia. (2018, July 19). Activity cost driver. Retrieved September 17, 2018, from http://www.investopedia.com/terms/a/activity-cost-driver.asp"

"The balance sheet has a close relationship with the company's A. mortgage. B. income statement. C. fair market value. D. statement of financial position." "

" "B Income statement. The balance sheet has a close relationship with the company's income statement. Changes to the income statement directly affect the numbers on the balance sheet. The balance sheet shows the company's mortgage as a liability, but the two are not closely related. The company's value on the balance sheet is quite different from its fair market value. Statement of financial position is another name for the balance sheet. SOURCE: FI:093 SOURCE: LAP-FI-010—The Right Balance (The Nature of Balance Sheets)"

"Which of the following is a characteristic of a profit-and-loss statement: A. Is the same as a balance sheet B. Summarizes expenses and revenue from sales C. Shows the owner's financial position D. Lists assets and liabilities" "

" "B Summarizes expenses and revenue from sales. A profit-and-loss statement is a business's financial picture that lists all revenue and expenses for a certain time period. A profit-and-loss statement is not the same as a balance sheet. Characteristics of a balance sheet include listing assets and liabilities and showing the owner's financial position. SOURCE: FI:094 SOURCE: LAP-FI-004—Watch Your Bottom Line (Income Statements)"

"Ethan is developing common-size financial statements so that he can compare financial performance across several different companies. Ethan is conducting __________ analysis. A. horizontal B. vertical C. ratio D. trend" "

" "B Vertical. Three common types of financial statement analysis are vertical analysis, horizontal analysis, and ratio analysis. When conducting vertical analysis, an individual calculates each individual item on a financial statement as a percentage of the total. After doing so, the analyst can develop common-size financial statements that allow him/her to compare performance across several years for a single company or across several different companies. Horizontal analysis, which is sometimes known as trend analysis, involves analyzing dollar amounts on financial statements for a single company across several years. Horizontal analysis does not involve the development of common-size financial statements. Ratio analysis involves the calculation of ratios based on a company's financial data. It allows analysts to compare numbers within a financial statement and determine the relationships among numbers on different financial statements. It can also be used to compare the performance of multiple companies. SOURCE: FI:334 SOURCE: Investopedia. (2018, June 13). Vertical analysis. Retrieved September 17, 2018, from https://www.investopedia.com/terms/v/vertical_analysis.asp"

"Jana noticed a problem while reviewing her company's monthly income statement. She verified that the total revenue was $4,590 and the total expenses were $1,452. However, the income statement showed a net income total of $1,383. Which of the following reflects the correct net income: A. $3,381 B. $5,973 C. $3,138 D. $1,833" "

" "C $3,138. A business's income statement, or profit-and-loss statement, is a financial summary that shows how much money the business has made or has lost over a specific period of time. A typical income statement includes revenue, operating expenses, and net income (profit). To determine the net income, subtract the total expenses from the total revenue ($4,590 - $1,442 = $3,138). It is important to verify the income statement's accuracy because businesses use the information to make important decisions about the business. If the information is incorrect (e.g., transposed numbers) and errors are not corrected, the business may make unwise decisions on the basis of faulty information. SOURCE: FI:335 SOURCE: Investopedia. (2018, August 9). Net income. Retrieved September 17, 2018, from https://www.investopedia.com/terms/n/netincome.asp"

"An example of an operating expense that is included in a business's income statement is A. inventory. B. purchases. C. commissions. D. allowances." "

" "C Commissions. Commission is a percentage of the total sale amount paid to the individual who makes the sale. Many businesses pay a commission to salespeople, which is part of their salary. Salaries and wages are operating expenses that are listed in a business's income statement. Purchases and value of inventory are included in the cost of merchandise sold category of an income statement. Allowances are deductions that are indicated in the revenue category of an income statement. SOURCE: FI:102 SOURCE: Kennon, J. (2017, May 29). Operating expense on the income statement. Retrieved September 19, 2017, from https://www.thebalance.com/operating-expense-on-the-income- statement-357586"

"An example of an internal change that could affect a business's sales forecast is a change in the A. length of a national recession. B. number of competitors in the market. C. size of the sales force. D. levels of consumer spending." "

" "C Size of the sales force. An internal change is one that occurs within the business and is under the business's control. Increasing or decreasing the number of people on the sales force is an internal change that will affect the sales forecast. For example, decreasing the size of the sales force may cause sales to fall because the remaining staff may not be able to handle as many sales. Changes in the number of competitors in the market, the length of a national recession, and the levels of consumer spending are external changes. SOURCE: FI:096 SOURCE: Freelancer. (2015, April 14). Sales forecasting: Factors to consider. Retrieved September 17, 2018, from https://www.freelancer.com/community/articles/sales-forecasting-factors-to- consider"

"Which of the following is an example of an intangible asset: A. Equipment B. Inventory C. Office building D. Accounts receivable" "

" "D Accounts receivable. Accounts receivable are money that is owed to a business. They are an asset to the business, but they do not have a physical presence. Inventory, buildings, and equipment are all tangible assets—they have a physical presence. SOURCE: FI:093 SOURCE: LAP-FI-010—The Right Balance (The Nature of Balance Sheets)"

"Which of the following phrases effectively describes a business's income statement: A. Income preview B. Budget estimate C. Cash-flow report D. Financial picture" "

" "D Financial picture. A business's income statement summarizes where the business's money came from and where it went. It is an entire financial picture that lists all revenue and expenses for a certain time period, usually one year. An income statement includes information used in budgets and cash-flow reports. It is a picture of a business's financial status, rather than a prediction of the financial future. SOURCE: FI:094 SOURCE: LAP-FI-004—Watch Your Bottom Line (Income Statements)"

"Marginal analysis is a cost/benefit decision process that seeks to determine A. the best qualities to look for in potential employees. B. how a manager can cut costs. C. the likely returns on a new investment. D. profit-maximizing quantity of output." "

" "D Profit-maximizing quantity of output. Marginal analysis compares marginal cost and marginal revenue to determine the amount of output that produces the greatest profits. It does not help determine how a manager can cut costs. Although it might help an investor to choose among investments, it does not determine the likely returns on a particular, new investment. And, although it might help determine the optimal number of employees, it does not determine the best qualities to look for in potential employees. SOURCE: FI:358 SOURCE: Investopedia. (2015, June 12). Marginal analysis. Retrieved September 19, 2017, from http://www.investopedia.com/terms/m/marginal-analysis.asp"

"Why do lending institutions carefully evaluate how well a business meets certain criteria before making a loan? A. To improve relations B. To provide service C. To prevent theft D. To reduce risk" "

" "D To reduce risk. There is a risk involved in making loans because a business may default and be unable to repay the bank. To reduce the risk, lending institutions use certain criteria to evaluate if the business is creditworthy. For example, a bank would run a credit check to find out if the business pays its bills on time. Also, a bank would review the business's financial records to determine if it earned sufficient income to pay expenses and also repay the loan. By carefully evaluating how well a business meets certain criteria, a bank is able to reduce the risk involved in making a loan. Lending institutions do not evaluate businesses to provide service, prevent theft, or improve relations. SOURCE: FI:034 SOURCE: Business Filings. (2012, May 24). What banks look for when reviewing a loan application. Retrieved September 17, 2018, from https://www.bizfilings.com/toolkit/research- topics/finance/business-finance/what-banks-look-for-when-reviewing-a-loan-application"

"Which of the following financial analysis methods is used to show the relationship of each component to the total within a single financial statement: A. Fundamental analysis B. Horizontal analysis C. Industry analysis D. Vertical analysis" "

" "D Vertical analysis. Vertical analysis is often used to show the relationship of each component to the total within a single financial statement. Horizontal analysis is a comparison of the same items on a company's financial statements for two or more periods. Businesses often compare their financial results with industry standards or averages to determine how well they are actually performing. Fundamental analysis involves studying all aspects of a company in an effort to understand its intrinsic value. SOURCE: FI:334 SOURCE: CCD Consultants. (2009-2015). Financial ratios interpretation. Retrieved September 19, 2017, from http://www.ccdconsultants.com/calculators/financial-ratios/financial-ratios- analysis-and-interpretation?tab=interpretation"

"A business's balance sheet lists $2,500 in cash, $6,125 in accounts receivable, $3,775 in inventory, $10,350 in machinery and equipment, and $4,280 in accounts payable. Calculate the business's total assets. A. $22,750 B. $18,470 C. $15,225 D. $27,030"

"A $22,750. A business's balance sheet shows the business's financial condition at a certain point in time. It includes all assets, debts, and the owner's equity. Total assets include items such as cash, accounts receivable, inventory, machinery and equipment, buildings, land, investments, and interest. Total assets do not include accounts payable, which are considered debts. To calculate the business's total assets, add the figures for cash, accounts receivable, inventory, and machinery and equipment ($2,500 + $6,125 + $3,775 + $10,350 = $22,750). SOURCE: FI:093 SOURCE: Farese, L.S., Kimbrell, G., & Woloszyk, C.A. (2009). Marketing essentials (p. 772). Woodland Hills, CA: Glencoe/McGraw-Hill."

"Patrick wants to keep a certain amount of his money in a safe place in which he can earn interest on his savings, and also be able to withdraw his funds without experiencing sizable financial penalties. In what type of financial institution should Patrick consider placing his money? A. A retail bank B. The stock market C. A retirement fund D. A holding company"

"A A retail bank. Retail banks provide a variety of financial services, including savings and checking accounts for individuals. Opening a savings account with a properly insured bank will provide Patrick with a safe place to keep his money, and he will earn interest on the money that he places in his savings account. Patrick will also be able to remove money to pay bills without incurring the financial penalties he might experience if he borrowed money from his retirement fund. Investing in the stock market involves risk; however, Patrick may earn dividends on stocks that perform well. A holding company (parent company) is a business that owns the majority of another company's shares of stock. SOURCE: FI:075 SOURCE: Investopedia. (2018, January 26). Retail banking. Retrieved September"

"Which of the following is an example of legal tender issued by the federal government: A. Bank notes B. Corporate bonds C. Mutual funds D. Municipal bonds"

"A Bank notes. Bank notes are currency. Currency is the paper or metal (coin) money that individuals and businesses use to buy and sell goods and services. Paper money and coins are issued by a country's government. Corporate bonds are bonds issued by corporations to fund operating expenses. A mutual fund is a collection of shareholders' money that is invested by professional fund managers in an assortment of different securities, such as stocks and bonds. A bond is a piece of paper (real or virtual) that says the governing body or corporation will borrow your money at a particular interest rate for a particular period of time. Municipal bonds are issued by local governments. SOURCE: FI:059 SOURCE: McMahon, M., & Harris, B. (2018, August 27). What is a bank note? Retrieved September 13, 2018, from http://www.wisegeek.com/what-is-a-bank-note.htm"

"In an ancient society, townspeople used cows to pay for the things they needed. The cows are an example of __________ money. A. commodity B. representative C. fiat D. paper"

"A Commodity. Commodity money is a physical item that has value on its own. A cow, for example, has value even if it is not being used as money. Fiat money is not backed by a commodity. Instead, it has value because the government says it does. Representative money can be traded in for something valuable. A cow is not an example of paper money. SOURCE: FI:059 SOURCE: Money Instructor. (2002-2017). What is money? The evolution of money. Retrieved September 6, 2017, from http://www.moneyinstructor.com/doc/moneyevolution.asp"

"Individuals often gain wealth from saving money in a retirement account because A. employers sometimes match employees' contributions. B. retirement accounts guarantee future financial security. C. many governments offer loan relief as a reward for saving. D. retirement account holders can accumulate rewards points."

"A Employers sometimes match employees' contributions. Many employers will match an employee's contribution to his/her savings account up to a certain amount each year. As a result, individuals essentially earn "free money" for their retirement, simply by saving. Although contributing to a retirement account is a good way to ensure future financial health, it does not guarantee financial security. Governments do not typically offer loan relief as a reward for saving. Retirement account holders do not accumulate rewards points for saving. SOURCE: FI:270 SOURCE: CNN Money. (2018). What is a matching contribution? Retrieved September 13, 2018, from http://money.cnn.com/retirement/guide/401k_basics.moneymag/index9.htm"

"Bryant recently applied for a personal loan from the Maryville Community Credit Union, but his application was rejected. Bryant has a friend who works at the credit union, so he asks her what went wrong with his application. She told Bryant that the credit union's manager is prejudiced and rarely accepts applications from individuals of Bryant's religion. The credit union violated Bryant's right to A. equal credit opportunity. B. a guaranteed credit line. C. proper notice of rejection. D. practice his religion."

"A Equal credit opportunity. All individuals are entitled to receive equal opportunity for credit under consumer credit protection laws. This means that creditors may not discriminate against an individual because of his/her race, color, religion, national origin, sex, marital status, or age when they make credit decisions. Individuals are not entitled to a credit guarantee. Methods of notifying candidates of their rejection for credit lines vary, and there is not a single "proper" way that is considered appropriate. Maryville Credit Union discriminated against Bryant because of his religion, but it did not violate his right to practice his religion. SOURCE: FI:002 SOURCE: Irby, L. (2018, April 21). Consumer credit laws you should be familiar with. Retrieved September 10, 2018, from https://www.thebalance.com/consumer-credit-laws-you-should-be-familiar-with-4068374"

"Chandra wants to develop a personal budget. What is the first step she should take? A. Establishing her goals B. Estimating her income C. Determining regular expenditures D. Allocating income"

"A Establishing her goals. Before Chandra figures out the details of her budget, she needs to establish her goals. She can't create an effective or realistic budget unless she knows what goals she's focusing on. Estimating income, determining regular expenditures, and allocating income occur later in the budget- development process. SOURCE: FI:066 SOURCE: Clark. (2017, March 23). How to create a budget, track your spending and reach your goals. Retrieved September 12, 2017, from http://clark.com/story/tracking-spending-best-way- maintain-budget/"

"Morgan received an email that claimed to be from her credit card company. The email said that her account would be cancelled immediately if she didn't follow a link and enter her credit card and social security numbers. After Morgan did so, she found out that the email wasn't actually from her credit card company. Instead, it was a scam designed to steal her personal information. Morgan was a victim of A. phishing. B. baiting. C. skimming. D. hacking."

"A Phishing. Phishing occurs when a scammer sends out an email pretending to be a legitimate company, such as a credit card company, in order to gain personal information. The phisher typically sends readers to a fake website that asks them to enter personal information, which can include credit card numbers, social security numbers, bank account numbers, etc. It's a good idea to be especially suspicious of any requests for personal information you receive online. Avoid giving out personal information unless you contact the company yourself. Skimming occurs when a thief runs a credit card through a ""skimmer"" to steal personal information. Hacking involves a person gaining access to a computer system to steal information. Baiting is not a term commonly used to describe phishing. SOURCE: FI:073 SOURCE: Microsoft Safety & Security Center. (2017). How to recognize phishing email messages, links, or phone calls. Retrieved September 6, 2017, from https://www.microsoft.com/en- us/safety/online-privacy/phishing-symptoms.aspx"

"The following is written on the back of a check: "For deposit only; Maria Evans; Account #285655." This is an example of a(n) __________ endorsement. A. restrictive B. general C. open D. congruent"

"A Restrictive. Banks require checks to be endorsed before processing. Endorsement involves signing the back of the check, which allows the funds to be transferred from the check source to the payee. There are different types of endorsements. A restrictive endorsement limits how the check can be processed. In the example, the check can only be deposited to the endorser's (Maria) specified account. The check cannot be cashed or placed into another account. A general endorsement allows the endorser to cash the check or deposit it into the account of his/her choice—there are no restrictions placed on the funds transfer. Open and congruent endorsements are fictitious terms. SOURCE: FI:560 SOURCE: Pritchard, J. (2018, August 2). See how to endorse checks. When and how to sign. Retrieved September 10, 2018, from https://www.thebalance.com/how-to-endorse-checks-315300"

"Iris buys an expensive new car using installment credit. Although the __________ for the car is made out in her name, the company that sold the car has a __________ on it until Iris makes all the installment payments. A. title; lien B. title; principal C. principal; lien D. interest; principal"

"A Title; lien. Installment credit is commonly used to purchase large, expensive items such as furniture or cars. The title, or legal ownership document, for an item purchased on installment credit is made out in the name of the credit user (in this case, Iris). However, the seller (the car company) has a lien, or claim, on the title until all installment payments have been made. If Iris neglects to pay, the company can repossess the car. Interest is the fee that the lender charges the borrower for the use of credit. The principal is the original amount of a loan on which the amount of interest is based. SOURCE: FI:002 SOURCE: LAP-FI-002—Give Credit Where Credit Is Due (Credit and Its Importance)"

"Why do some governments require businesses to use specific accounting methods? A. To provide external sources with accurate financial information B. To ensure that the businesses are complying with intellectual property laws C. To control the ways in which businesses develop credit policies D. To monitor how businesses establish their sales goals and budgets"

"A To provide external sources with accurate financial information. The accounting function collects and records the financial information in the appropriate format. Governments require businesses to provide certain types of financial information to regulatory agencies and shareholders in specific ways. For example, some governments require large businesses to use a specific accounting method, such as the accrual method. The accrual accounting method records transactions at the time they occur even if no money changes hands at that time. An advantage of using the accrual method is that it provides investors, creditors, and customers with a more accurate picture of a business's cash flow over time. Governments do not require businesses to use a specific accounting method to ensure that they are complying with intellectual property laws, to control the ways in which businesses develop credit policies, or to monitor how businesses establish sales goals and budgets. SOURCE: FI:353 SOURCE: Fishman, S. (2017). Cash vs. accrual accounting. Retrieved September 6, 2017, from http://www.nolo.com/legal-encyclopedia/cash-vs-accrual-accounting-29513.html"

"Margot wants to be sure that when she dies, her money, house, and cars will go to her sister. Margot should create a A. will. B. retirement fund. C. college savings fund. D. bank account."

"A Will. A will is a legal document that outlines how a person wishes his/her assets to be distributed once s/he dies. If Margot wants to be sure that her sister receives her assets, she should set up a will. Retirement funds, college savings funds, and bank accounts do not guarantee that Margot's assets will be given to her sister. SOURCE: FI:064 SOURCE: Williams, G. (2018, June 19). 10 steps to writing a will. Retrieved September 14, 2018, from https://money.usnews.com/money/personal-finance/family-finance/articles/steps-to-writing-a- will "

"Talia manages a business's accounts payable, accounts receivable, inventory, and cash. Talia is responsible for A. working capital management. B. capital investment decisions. C. the business's capital structure. D. market risk management."

"A Working capital management. An organization's finance function is responsible for managing the business's working capital and making capital investment decisions for the company. Working capital is the difference between a business's current assets and current liabilities. Working capital management focuses on the company's current balance of assets and liabilities and involves the management of accounts payable and receivable, inventory, and cash. Working capital management involves decisions made for the short-term—one year or less. Capital investment decisions determine which projects the business will invest in, how the investment(s) will be financed, and whether or not to pay dividends to the company's shareholders. The business's capital structure consists of the mix of debt and equity financing used to finance investments and projects. Market risk is the risk of financial loss due to the decreased value of an investment, and market risk management involves using financial instruments to manage exposure to market risk. Capital investment decisions, the business's capital structure, and market risk management are typically long-term in nature. Talia is not responsible for capital investment decisions, the business's capital structure, or market risk management. SOURCE: FI:354 SOURCE: LAP-FI-007—Money Matters (Role of Finance)"

"Which of the following is an example of a business using cash-control procedures: A. Writing checks for payments B. Marking all currency C. Eliminating expense reimbursement D. Accepting credit cards"

"A Writing checks for payments. Controlling cash also involves the payment of cash for purchases. To control cash, businesses usually write checks to pay invoices, and for other purchases and expenses. The checks draw on the business's cash; however, writing checks is safer than paying with cash because there is a record of the transaction. Businesses do not mark currency. Eliminating expense reimbursements is not a cash-control procedure. Businesses accept credit cards as a convenience to customers rather than to control cash. SOURCE: FI:113 SOURCE: Tatum, M., & Harris, B. (2018, August 27). What is cash control? Retrieved September 13, 2018, from https://www.wisegeek.com/what-is-cash-control.htm"

"A coffee shop wants to appeal to young, technology-savvy consumers. To appeal to this demographic, the business should consider A. instituting a minimum amount for credit card payments. B. accepting mobile credit card payments. C. gathering analytics from credit card payments. D. purchasing a receipt printer."

"B Accepting mobile credit card payments. Businesses have a lot to consider when deciding whether or not to accept credit card payments—and how to accept credit card payments. Mobile credit cards are increasingly popular with the younger, technologically-savvy demographic, so the coffee shop should consider accepting mobile credit card payments. Instituting a minimum amount for credit card payments, gathering analytics from credit card payments, and purchasing a receipt printer are not necessarily related to appealing to a young, tech-savvy demographic. SOURCE: FI:789 SOURCE: Worldpay. (2018). Need to accept credit cards? 5 critical considerations. Retrieved September 12, 2018, from https://www.vantiv.com/merchant-services/choose-merchant-card- processing"

"When the unemployment rate rises, stock values typically A. increase. B. decrease. C. stabilize quickly. D. remain unchanged."

"B Decrease. Economic factors such as the unemployment rate can affect stock prices. If trends indicate a rise in unemployment rates, the economy may be heading into a recession. In this situation, individuals are often fearful about losing their jobs and are likely to hold onto their money rather than invest it. They may also sell their stock in anticipation of needing the money for living expenses. Businesses often react the same way—they may hold onto their funds until the economy rebounds before making major investments. All these factors can cause the value of stocks to decrease. SOURCE: FI:574 SOURCE: Levitt, A. (n.d.). Economic indicators that affect the U.S. stock market. Retrieved September 11, 2018, from http://www.investopedia.com/articles/investing/031413/economic-indicatiors- affect-us-stock-market.asp"

"Balancing the business's objective to make a profit with the interests of all the business's stakeholders is a primary consideration related to the business's A. creative processes. B. financial ethics. C. maintenance activities. D. forecasting methods."

"B Financial ethics. An important aspect of financial ethics is making sure the business makes a profit and serves the best interests of its stakeholders (e.g., employees, customers, investors). This would include honest communication, transparent financial reporting, and fair pricing practices. Creative processes, maintenance activities, and forecasting methods are important business considerations; however, these activities are not directly related to the business's profitability and the stakeholders' best interests. SOURCE: FI:355 SOURCE: Peavler, R. (2018, April 26). How financial ethics and profitability are connected. Retrieved September 13, 2018, from https://www.thebalancesmb.com/how-financial-ethics-and- profitability-are-connected-393247"

"Sasha's personal budget contains two broad categories for her expenses. She should put her mortgage, her car insurance payment, and her utilities in the __________ expenses category. A. variable B. fixed C. net D. gross"

"B Fixed. When you make a personal budget, it's important to divide your expenses into two broad categories. Fixed expenses are those that stay the same from month to month, like mortgages, car insurance payments, and utilities. Variable expenses change from month to month. Net and gross are terms that are used to describe income, not expenses. SOURCE: FI:066 SOURCE: Inc. (2018, April 19). Fixed and variable expenses. Retrieved September 10, 2018, from http://www.inc.com/encyclopedia/fixed-and-variable-expenses.html"

"Why do many companies set a floor limit specifying a maximum amount a customer is allowed to charge to a credit card? A. To increase the amount of sales B. To protect the company against fraud C. To encourage the use of credit cards D. To prevent fraud from occurring"

"B To protect the company against fraud. A floor limit protects a business from excessive charges made illegally because the business is only held liable for the amount of the floor limit. Setting a floor limit does not create any buying incentive that would increase the amount of sales or encourage the use of credit cards. Setting a floor limit does not prevent fraud, but it can help reduce its impact. SOURCE: FI:789 SOURCE: Farese, L.S., Kimbrell, G., & Woloszyk, C.A. (2012). Marketing essentials 2012 (pp. 367- 369). Columbus, OH: Glencoe/McGraw-Hill."

"Finemart's bookkeeper journalizes the business's income and expenditures at the time they occur even if no money changes hands at that time. Finemart uses the __________ accounting method to record transactions. A. managerial B. cash C. accrual D. tax"

"C Accrual. Businesses using the accrual accounting method journalize income and expenditures at the time they occur even if no money changes hands at that time. This means that the business enters the amount of a transaction into the appropriate journal when a customer makes a credit purchase, or when the business orders goods from a supplier. Businesses using the cash accounting method record income and expenditures at the time the money changes hands. This means that the business enters the amount of a transaction into one of its journals on the day the money is received from a customer or paid out to a creditor. Managerial accounting involves reporting financial data to internal users. Tax accounting involves recording transactions for tax purposes. Finemart does not use the cash accounting method, and there is no indication that it is using managerial accounting or tax accounting to record transactions, either. SOURCE: FI:085 SOURCE: LAP-FI-085—Show Me the Money (Nature of Accounting)"

"Which of the following statements regarding debt and equity markets is correct: A. If a company suffers financial difficulties, its stockholders are paid before its bondholders. B. Investors who purchase a company's bonds gain partial ownership of the business. C. Bonds, which are debt instruments, are typically less risky investments than stocks. D. Common stock, an example of a debt instrument, is traded on the equity market. "

"C Bonds, which are debt instruments, are typically less risky investments than stocks. Equity instruments such as common stock are traded on the equity market. Individuals who purchase equity instruments, not debt instruments such as bonds, gain partial ownership of a business. If a company suffers financial difficulties, its bondholders are paid before its stockholders. SOURCE: FI:337 SOURCE: Scrofano, J. (2017, September 26). Debt and equity instruments. Retrieved September 14, 2018, from https://bizfluent.com/list-7178096-debt-equity-instruments.html"

"Saving the money that you might ordinarily spend each day on a cup of coffee and a donut A. is the sacrifice you must make for future financial security. B. will not result in significant savings. C. can help you achieve larger financial goals. D. is not a practical step in reaching your financial goals."

"C Can help you achieve larger financial goals. Making purposeful spending decisions, instead of spending out of habit or impulsively, is an important step in achieving larger financial goals. Even small daily savings add up to significant amounts over time. Day-to-day spending does not have to be sacrificed entirely for future financial security, as long as you're aware of your spending and are making purposeful decisions. Small savings every day are a very practical step in reaching your financial goals. SOURCE: FI:065 SOURCE: Benve, R. (2018, August 14). How to set SMART personal financial goals. Retrieved September 10, 2018, from https://toughnickel.com/personal-finance/How-to-Set-Personal- Financial-Goals-Budgeting"

"When John was injured in an auto accident on his way to work, what kind of insurance paid his hospital bill? A. Term B. Disability C. Health D. Life"

"C Health. Health insurance covers such medical expenses as hospital bills, doctors' fees, lab charges, and other medical costs. Disability insurance is sometimes called loss-of-income insurance because it pays a percentage of a worker's normal wages to the worker when s/he cannot work because of an illness or injury. Life insurance pays a sum of money at the time of death. Term is a form of life insurance that covers a specific number of years. SOURCE: FI:081 SOURCE: Kapoor, J.R., Dlabay, L.R., & Hughes, R.J. (2009). Personal finance (9th ed.) [pp. 343-344]. New York: McGraw-Hill Irwin."

"Income from sales, cost of goods sold, gross profit on sales, operating expenses, and net profit or loss are all part of a(n) A. sales budget. B. balance sheet. C. income statement. D. marketing plan."

"C Income statement. An income statement is a financial summary that shows how much money the business has made or lost. It is also called a profit-and-loss statement. A balance sheet shows a firm's assets, liabilities, and owner's equity. A marketing plan is a set of procedures or strategies for attracting the target customer to the business. A sales budget is an estimate of expenditures related to sales. SOURCE: FI:094 SOURCE: LAP-FI-004—Watch Your Bottom Line (Income Statements)"

"Individuals have the right to provide additional information about their income status and payment history when A. estimating their tax liability. B. opening a checking account. C. validating their credit history. D. hiring a financial planner."

"C Validating their credit history. Individuals often validate their credit history before applying for additional credit to make sure that the information in the history is accurate. In some cases, individuals may want to provide additional information to more fully explain their income status if the credit history does not include all sources of income. Also, if there was a history of late payments for a certain account, individuals might want to explain that the charges were in dispute and that the matter has been resolved. Adding additional information often helps to answer questions related to an individual's credit history. Individuals usually do not provide additional information about their income status and payment history when opening a checking account, estimating their tax liability, or hiring a financial planner. SOURCE: FI:072 SOURCE: Well Traveled Mile. (2015, June 10). What to do if your credit card application needs further review. Retrieved September 12, 2017, from https://welltraveledmile.com/what-to-do-if-your- credit-card-application-needs-further-review/"

"Which of the following international financial institutions provides financing and advice to countries to encourage economic development: A. Commonwealth of Nations B. Small Business Administration C. World Bank Group D. Liberty Institute"

"C World Bank Group. The World Bank Group, which includes the World Bank as well as several other international organizations, is an international financial institution that provides financing and advice to impoverished countries to encourage economic development and eliminate poverty. The Small Business Administration is a U.S. government financial institution that provides financial assistance and advice to American small businesses. The Commonwealth of Nations and the Liberty Institute are not financial institutions. The Commonwealth of Nations is an international organization that promotes world peace, democracy, and free trade. The Liberty Institute is an international organization based in the country of Georgia that advocates civil liberties as well as public accountability and effective governance. SOURCE: FI:336 SOURCE: Investopedia (n.d.) The world bank. Retrieved September 14, 2018, from https://www.investopedia.com/terms/w/worldbank.asp "

"Which of the following has contributed to financial globalization: A. An increase in restrictions on foreign investment and international trade B. An increase in taxes on international financial transactions C. A reduction in foreigners' access to domestic financial markets D. A movement from fixed exchange rates to floating exchange rates "

"D A movement from fixed exchange rates to floating exchange rates. A fixed exchange rate is one that the government sets and maintains. A fixed exchange rate does not change very often. A floating exchange rate, on the other hand, is constantly in flux—determined by the market through supply and demand. To enable capital to flow more freely across their national borders, countries around the world have moved from fixed to floating exchange rates, lowered taxes on international financial transactions, increased foreigners' access to domestic financial markets, and decreased restrictions on foreign investment and international trade. SOURCE: FI:575 SOURCE: Pologeorgis, N. (n.d.). The globalization of financial services. Retrieved September 14, 2018, from https://www.investopedia.com/articles/financial-theory/09/risk-free-rate-return.asp"

"A business comparing the monthly bank statement with the entries in its checkbook is an example of a(n) A. break-even analysis. B. accounts-payable system. C. money-handling technique. D. cash-control procedure."

"D Cash-control procedure. Businesses develop procedures to control cash in order to prevent loss. One procedure involves carefully comparing the monthly bank statement with the entries in the business's checkbook. If a business discovers errors or discrepancies, it should review the statement with the bank to identify the source of the problem. In some cases, banks accidentally debit the wrong account or post deposits to the wrong account. If the business fails to detect the error, it may lose those funds. Accounts payable are all the monies owed by the business to others. Money handling involves accepting cash, counting change, etc. Break-even analysis is a financial analysis whose purpose is to identify the level of sales needed to reach the break-even point at various prices. SOURCE: FI:113 SOURCE: Ann, L. (2009, April 30). Importance of cash control. Retrieved September 12, 2017, from http://ezinearticles.com/?Importance-Of-Cash-Control&id=2287169"

"The Money for You Bank recently merged with the Keeping You Safe Insurance Company. The merger is an example of __________ in the finance industry. A. licensing B. consolidation C. tactical planning D. convergence"

"D Convergence. Convergence occurs when financial providers from different financial sectors (e.g., banking services and insurance) merge. Consolidation takes place when financial providers within the same institutional category merge. For example, if two banks were to merge, they would be consolidating their companies. Tactical planning is short-range planning (one year) of specific actions the business will take. Two businesses should look much further ahead than just one year before making the decision to merge. Licensing is a business structure that requires the authorization or permission from an owner of another entity to use trademarked, copyrighted, or patented material for a specific activity, during a specific time period, for the profit of both parties. Businesses that enter into licensing agreements do not merge. SOURCE: SOURCE: FI:573 Abel, S., & Back, C. (2016). Achieving convergence of finance, risk and actuarial functions: Beyond transformations. Retrieved September 14, 2018, from http://www.ey.com/Publication/vwLUAssets/ey-achieving-convergence-of-finance-risk-and- actuarial-function-beyond-transformation/$FILE/ey-achieving-convergence-of-finance-risk- and-actuarial-function-beyond-transformation.pdf"

"Saving money helps you achieve __________ goals, while investing money helps you achieve __________ goals. A. long-term; career B. long-term; short-term C. career; short-term D. short-term; long-term"

"D Short-term; long-term. Saving money involves putting away a portion of your income to use in the future. This may involve placing money in a savings account where you can easily withdraw the funds when you need them. Saving money can help you achieve your short-term goals such as buying a new computer or taking a trip. Often, people use their savings to make investments. Investing money involves using money to make money. By investing your money, you can make more money than you can with just income and savings. The money you make from your investments can help you achieve your long-term goals such as funding your children's college education, starting your own business, or using it for living expenses when you retire. Your career goals involve your vocation, occupation, or profession and do not always involve saving or investing money. SOURCE: FI:270 SOURCE: The Money Advice Service. (n.d.). Should I save or invest my money? Retrieved September 6, 2017, from https://www.moneyadviceservice.org.uk/en/articles/should-i-save-or-invest"

"Which of the following investments typically carries moderate risk: A. Savings accounts B. Collectibles C. Certificates of deposit D. Stocks"

"D Stocks. Some securities and investments are riskier than others. Low-risk investments are typically very stable, meaning that there's little chance of financial loss. However, that also means that there's little chance of substantial financial gain, either. Low-risk investments include savings accounts, money market accounts, certificate of deposits, and bonds. High-risk investments, on the other hand, provide a much greater opportunity for loss or return. Collectibles are high-risk investments—they may skyrocket or plummet in value depending on demand. Finally, moderate-risk investments fall somewhere between low-risk and high-risk investments in terms of loss and return. Stocks, mutual funds, and real estate typically carry moderate risk. SOURCE: FI:077 SOURCE: LAP-FI-077—Invest for Sucess (Types of Investments)"

"Each pay period, Kendra has her employer automatically deposit her paycheck into her checking account so the funds are available for use when she needs them. In this situation, money is functioning as a(n) A. form of cash. B. unit of measure. C. medium of exchange. D. store of value."

"D Store of value. Money functions as a store of value when it is saved rather than spent. The money can be held over time, and it retains value for future purchasing power. Money serves as a unit of measure when it is used to express the value or worth of something. Money serves as a medium of exchange when it is used to obtain goods and services. Cash is a form of money—currency and coins. Because Kendra's paycheck was automatically deposited in her bank account, currency and coins have not been touched. SOURCE: FI:060 SOURCE: Clark, B., Basteri, C.G., Gassen, C., & Walker, M. (2014). Marketing dynamics (3rd ed.) [p. 45]. Tinley Park, IL: The Goodheart-Willcox Co."

"What type of insurance do you need to purchase if you want to ensure that your beneficiaries receive an inheritance? A. Long-term care insurance B. Term life insurance C. Disability insurance D. Universal life insurance"

"D Universal life insurance. With universal life insurance, you will be insured for life and be able to grow your savings tax-free. This coverage is adapted to meet your insurance needs at any stage of life. It also lets you build your savings so that your beneficiaries are left with an inheritance. Term life insurance provides coverage for a set period of time—one, five, 10, or 20 years. This insurance pays only if you are insured at the time of injury or illness. At the end of the coverage, you can renew your coverage, convert the term life insurance into permanent coverage, or let the policy lapse. Disability insurance pays a nontaxable benefit every month that you are disabled. Its coverage ends when your disability ends and does not include an inheritance for beneficiaries. Long-term care insurance is needed if you lose your independence due to physical or mental limitations following an accident or illness. SOURCE: FI:081 SOURCE: Pareto, C. (2017, March 27). Intro to insurance: Types of life insurance. Retrieved September 12, 2017, from http://www.investopedia.com/university/insurance/insurance8.asp"

"Caitlin has three credit cards. She receives her credit card statement for one of the cards and has to decide how to pay the $500 she spent with that card. To pay the least for use of the credit, she should pay with __________ and make the __________ payment. A. cash; full B. cash; minimum C. another card; full D. another card; minimum"

A Cash; full. Paying the statement in full will assure that she does not have to pay extra interest on the balance and will help improve her credit history. Paying in cash is better for her credit history's health than paying with another card, because eventually she will have to pay the money owed on the other card. Paying for one card with another can lead to a downward spiral of credit card debt. SOURCE: FI:071 SOURCE: Evolve Money. (2014, August 13). 5 simple ways for college students to stay out of credit trouble. Retrieved September 13, 2017, from www.evolvemoney.com/2014/08/13/bad-creditcollege-students

"Which of the following is a user-owned, nonprofit, cooperative financial institution: A. Credit union B. Commercial bank C. Insurance company D. Savings and loan association/"

A Credit union. A credit union is a nonprofit financial cooperative set up to provide savings and credit services to its members, who are also the owners of the credit union. Although many credit unions are tied to a particular business or labor union, some are not affiliated with any certain entity or organization. Commercial banks, insurance companies, and savings and loan associations are not user-owned, and they typically are for-profit financial institutions. A commercial bank is a full-service bank offering a variety of banking services including checking and savings accounts, secured and unsecured loans, installment credit, and bank credit cards. While an insurance company may offer some banking services, it is likely to focus on the sale of insurance policies to individuals and businesses. A savings and loan association is a financial institution that offers savings accounts and makes loans to borrowers, most of which are for real estate purchases. SOURCE: FI:075 SOURCE: Kapoor, J.R., Dlabay, L.R., & Hughes, R.J. (2009). Personal finance (9th ed.) [pp. 143-146]. New York: McGraw-Hill Irwin.

"Using a credit card is A. like getting a free loan if you pay the balance each month. B. a good way to pay for impulsive purchases. C. an inexpensive way to borrow money for an extended period. D. a privilege reserved for working adults."

A Like getting a free loan if you pay the balance each month. When you pay the balance on a credit card each month, you are getting what amounts to a free loan. A credit card is not a good way to pay for impulsive purchases—impulsive purchases are usually a bad idea no matter how you pay for them. Credit cards are not an inexpensive way to borrow money for an extended period. Nowadays, easy access to credit cards is a growing problem. SOURCE: FI:065 SOURCE: US Bank. (2018). How credit cards work. Retrieved September 20, 2018, from https://www.usbank.com/credit-cards/how-credit-cards-work.html

"Althea just received her first paycheck at work. She's excited, but as she looks at her pay stub she gets confused. She wants to know how much money she made in this pay period, but there are so many different numbers. If Althea wants to know how much money will actually be deposited in her bank account, she should look at the A. net pay. B. gross pay. C. federal income tax. D. state tax./"

A Net pay. The net pay is the amount of money Althea will actually receive after all her withholdings have been taken out. This is the money that will be deposited in her bank account. The gross pay is the amount she earns before her withholdings are taken out. Federal income tax and state tax are money that Althea owes to the government. SOURCE: FI:068 SOURCE: Clearpoint. (2017). How to read your paycheck stub. Retrieved September 26, 2017, from http://www.clearpointcreditcounselingsolutions.org/how-to-read-your-pay-stub/

"Which of the following is a disadvantage of investing in a savings account: A. The rate of return doesn't keep up with inflation. B. A minimum balance is required to keep the account active. C. You must pay a penalty for withdrawing your money early. D. There is a high potential for loss./"

A The rate of return doesn't keep up with inflation. Savings accounts earn low rates of return—usually not even enough to keep up with inflation. This causes the investor's money to lose buying power. Savings accounts do not require a minimum balance or incur penalties for withdrawing money early. Savings accounts are very low-risk investments, especially since they are insured by the FDIC. SOURCE: FI:077 SOURCE: LAP-FI-077—Invest for Success (Types of Investments)

"Justine's rich uncle wants to give her $5,000 toward the purchase of a car. But since Justine doesn't plan to buy the car for at least another year, her uncle told her that she can have the money now, or he can wait and give her the money when she actually buys the car. Justine chooses to take the money now and deposit it in her savings account. After all, her deposit will yield 6% interest compounded annually. A year from now, her $5,000 will be worth $5,300. What financial concept does this scenario illustrate? A. Time-value of money B. Dividend reinvestment plan C. Capital budgeting D. Accrual accounting"

A Time-value of money. Due to the time-value of money, any certain amount of money (such as Justine's $5,000) is worth more now than later as a result of its earning potential. Rather than letting her uncle keep the $5,000 until next year, Justine is wise to take the money now and put it in a deposit-bearing savings account. That way, Justine can earn more money ($300, to be precise) on the money from her uncle. A dividend reinvestment plan (also known as a DRIP) is a method used by many investors to grow their investments more quickly. It involves using stock dividends to purchase more shares of that stock. Capital budgeting is the process that a firm's financial managers use to determine which projects to invest in. The accrual accounting method involves recording transactions at the time they occur even if no money changes hands at the time. SOURCE: FI:270 SOURCE: Carther, S. (2017, May 18). Understanding the time value of money. Retrieved September 19, 2018, from http://www.investopedia.com/articles/03/082703.asp#axzz2BZGrTRpM

"What is one thing you can do to protect yourself from becoming a victim of identity theft? A. Use unusual passwords rather than your birth date to access your bank accounts. B. Place your Social Security number or your social insurance number on all documents. C. Keep your passport with you at all times. D. Always pay with cash rather than credit cards or check./"

A Use unusual passwords rather than your birth date to access your bank accounts. Identity theft occurs when somebody uses another individual's name and personal information (e.g., credit cards) for fraudulent purposes. Identity theft can occur even when you take precautions. To reduce the risk of identity theft, use unusual passwords to access your personal bank accounts rather than common information such as your birth date. To further prevent identity theft, do not place your Social Security number or social insurance number on unnecessary documents. Many people do not have passports. Paying with cash is not always a good idea because cancelled bank checks serve as a receipt for payments. If you pay your bills by sending cash, the envelope may get lost, and you have no record of making the payment. In addition, using credit cards wisely can help you build a good credit report, which will help you obtain loans for expensive items (e.g., car) in the future. SOURCE: FI:073 SOURCE: Ryan, J.S. (2012). Personal financial literacy (2nd ed.) [pp. 304-305]. Mason, OH: SouthWestern Cengage Learning.

"Which of the following is an example of processing a transaction with a debit card: A. When Naomi purchased office supplies, she presented the cashier with a card that automatically withdrew funds from her business's checking account. B. Tyler provided his card number and expiration date to the online travel agency so he could obtain his airline tickets now and receive his bill for the tickets later. C. After Ava finished her meal at an upscale restaurant, she presented the server with a card that provided Ava with a special discount off her bill. D. Jayden purchased a new video game from Great Deal Electronics using a store-issued card given to him by his grandfather/"

A When Naomi purchased office supplies, she presented the cashier with a card that automatically withdrew funds from her business's checking account. A debit card is a common method of payment for goods and services. Issued by financial institutions (e.g., banks), a cardholder scans the card into a machine at the point of sale. The information is transmitted to the appropriate location, and the funds are electronically transferred from the cardholder's checking or savings account to the seller's account. Credit is the arrangement in which businesses and individuals can purchase now and pay later. Individuals and businesses can obtain the goods and services that they need right away, such as airline tickets, and then pay for the tickets when they receive their statements from their lending institutions. Some businesses offer membership or customer-loyalty cards, which provide their members or customers with product discounts. Gift cards are store-issued vouchers that a gift giver can purchase for another person to use to buy the store's products. SOURCE: FI:058 SOURCE: Farese, L.S., Kimbrell, G., & Woloszyk, C.A. (2012). Marketing essentials (pp. 365-369). Columbus, OH: Glencoe/McGraw-Hill.

"While George is unable to work because of a back injury, he receives a check every week for 60% of his usual salary. George's check is a benefit of his __________ insurance. A. cash-value B. disability C. liability D. bodily injury"

B Disability. Disability insurance is sometimes called loss-of-income insurance because it pays a percentage of a worker's normal wages to the worker when s/he cannot work because of an illness or injury. It is often purchased through or by an employer. Cash-value is a type of life insurance that pays out the stated value of the policy at the time of death. Liability insurance protects property owners if someone else is injured on or by their property. Bodily injury is a type of liability insurance that protects vehicle owners when they cause injury to others. SOURCE: FI:081 SOURCE: Pareto, C. (2017, March 27). Intro to insurance: Disability insurance. Retrieved September 13, 2017, from http://www.investopedia.com/university/insurance/insurance5.asp

"Jeff is a college student who just obtained his first credit card. Which of the following should he NOT do: A. Pay his bill on time B. Buy more with his card than he can afford C. Think of his card as a loan D. Track his spending habits/"

B Buy more with his card than he can afford. If Jeff does this, he will be headed toward mounting credit card debt. He should pay his bill on time to build good credit. Thinking of his card as a loan and tracking his spending habits will keep him from overspending. SOURCE: FI:071 SOURCE: College Board. (2017). How to use credit cards wisely. Retrieved September 26, 2017, from https://bigfuture.collegeboard.org/pay-for-college/paying-your-share/how-to-use-credit-cardswisely

"A business advertised new-product financing with a 10% down payment and a low interest rate with monthly payments spread over four years. What type of loan was the business offering? A. Single payment loan B. Installment loan C. Cash payment D. Revolving credit loan"

B Installment loan. Loans that must be paid back in regular installments over a certain amount of time at a specified rate of interest are installment loans. They are frequently used to purchase costly items, such as automobiles, furniture, and appliances. Single payment loans are loans that must be paid off in a lump sum at a designated time in the future, possibly three months. Revolving credit loans are loans that are repaid on a regular basis in at least the minimum required amount. Cash payments are made for the full amount at the time of purchase. SOURCE: FI:002 SOURCE: LAP-FI-002—Give Credit Where Credit Is Due (Credit and Its Importance)

"Who is usually responsible for analyzing a business's income statement? A. Customers B. Managers C. Cashiers D. Bookkeepers"

B Managers. The income statement is usually analyzed by people who are responsible for the financial status of the business. This includes managers who look at ratios to monitor operations and determine whether a company is running efficiently. They also use the income statement to monitor yearly profit activity. Customers usually do not review a business's income statements unless they are also stockholders in the business. Bookkeepers usually are responsible for compiling income statement information but not analyzing it. Cashiers accept payments from customers, but they are not responsible for analyzing a business's income statements. SOURCE: FI:094 SOURCE: LAP-FI-004—Watch Your Bottom Line (Income Statements)

"Putting $100 in a savings account today and earning 4% interest over the next year illustrates the concept of A. principal modification. B. the time value of money. C. opportunity cost. D. inflationary impact."

B The time value of money. The term "time value of money" refers to the increase of an amount of money as a result of interest or dividends earned. A basic illustration of the time value of money is money and the interest it earns in a savings account. Principal is the deposited amount, which is not modified. Opportunity cost is what you have to give up when you make a choice. In this case, the opportunity cost of depositing $100 is spending it on something else. Inflation has a negative impact on the time value of money. SOURCE: FI:062 SOURCE: Campbell, S.R. (2010). Foundations of personal finance: Teacher's edition (p. 266). Tinley Park, IL: Goodheart-Willcox Company.

"Which of the following is a disadvantage of a checking account: A. Overdraft protection B. Direct payroll deposit C. Account balance requirement D. Online transaction options"

C Account balance requirement. Financial institutions often require customers to maintain a certain amount of money in their bank accounts. The account holder may be charged fees if the minimal account balance requirements are not met. Direct payroll deposit, overdraft protection, and online transaction options are benefits or services that many types of checking-account products offer that customers may need to pay fees to obtain. SOURCE: FI:058 SOURCE: Zacks. (2018, May 10). Advantages and disadvantages of savings and checking accounts. Retrieved September 14, 2018, from https://finance.zacks.com/advantages-disadvantagessavings-checking-accounts-3843.html

"What source of credit offers its members small loans, installment credit, and low interest rates? A. Commercial banks B. Sales finance companies C. Credit unions D. Savings and loan associations"

C Credit unions. A credit union is a financial cooperative set up to provide savings and credit services to its members. These services include small loans, installment credit, and low interest rates. Commercial banks offer many financial services including checking and savings accounts, secured and unsecured loans, installment credit, and bank credit cards. They offer small and large loans, and their interest rates vary. A sales finance company is a business that purchases borrowers' contracts from sellers. Their interest rates are higher than the rates offered by credit unions. A savings and loan association is a financial institution that offers savings accounts and makes loans to borrowers. Most of their loans are for large-ticket items such as real estate. SOURCE: FI:002 SOURCE: LAP-FI-002—Give Credit Where Credit Is Due (Credit and Its Importance)

"A primary reason governments regulate business accounting practices is to A. balance the business's budget. B. certify aging accounts. C. stabilize tax revenue. D. ensure consistent reporting."

D Ensure consistent reporting. The accounting function is responsible for collecting and reporting the business's financial information. Governments require businesses to provide certain types of financial information to regulatory agencies and shareholders in specific ways. By requiring businesses to follow the same standards and rules, financial reports are prepared in a consistent way, so the information is clear to all people who read and work with financial information. The primary reason that governments regulate business accounting practices is not to certify aging accounts, stabilize tax revenue, or to balance the business's budget. SOURCE: FI:353 SOURCE: QuickMBA. (1999-2010). Financial accounting standards. Retrieved September 13, 2017, from http://www.quickmba.com/accounting/fin/standards/

"According to a company's most recent statement of cash flows, its cash flow from operations for the most recent year was $1,375,400. The net cash used by investing was an outlay of $690,000. The net cash spent on financing was $140,300. What was the company's free cash flow? A. $686,400 B. $549,700 C. $545,100 D. $830,300"

C $545,100. A company's free cash flow represents the funds available for internal growth and expansion. It also indicates how well the company is managing its money. A positive free cash flow is usually a sign that the company is practicing responsible money management, which can result in increased shareholder yield. A negative free cash flow, however, indicates that the company needs to look for more and/or other sources of funding. To calculate the free cash flow, we first need to determine the company's capital expenditures by adding together the net cash used by investing and the net cash used by financing ($690,000 + $140,300 = $830,300). Finally, subtract the total capital expenditures from the cash flow from operations ($1,375,400 - $830,300 = $545,100). So, the company brought in $545,100 more than it spent. SOURCE: FI:541 SOURCE: Vane, J. (2017, September 26). How to calculate free cash flow. Retrieved September 19, 2018, from https://bizfluent.com/how-2364074-calculate-free-cash-flow.html

"Two years ago, Tommy borrowed $12,500 from his bank to purchase a new vehicle. Unfortunately, he lost his job about a year ago, and he hasn't been able to make a payment on the loan for six months. As a result, the bank is taking steps to seize his automobile. This is called a A. forbearance. B. foreclosure. C. repossession. D. debt consolidation./"

C Repossession. When a borrower defaults (fails to pay for an extended period of time) on his/her loan, the lender—in this case, Tommy's bank—may choose to claim the collateral—Tommy's car—that was used to secure the loan. Tommy will no longer be expected to pay on the loan, but he also will no longer have his vehicle. Foreclosure is similar to repossession, but the term is used strictly to refer to the seizure of a mortgaged property, usually a house or other type of real estate. Forbearance is an agreement between a mortgagee and a mortgager to delay foreclosure proceedings. A debt consolidation involves taking out a new loan to pay off multiple other loans and debts. SOURCE: FI:568 SOURCE: Federal Trade Commission. (n.d.). Consumer information: Vehicle repossession. Retrieved September 26, 2017, from https://www.consumer.ftc.gov/articles/0144-vehicle-repossession\

"What is the co-signer of a bank loan legally responsible to pay if the borrower defaults on the loan? A. The interest amount owed only B. The principle amount owed only C. The balance of the loan owed D. The original amount of the loan/"

C The balance of the loan owed. When the borrower does not have the collateral or credit history to obtain the loan, the lender often requests that another person co-sign for the loan. A co-signer is responsible for paying back the loan if the borrower does not. The co-signer is legally responsible for paying the balance of the loan owed, including interest and principle. If the original loan was $5,000 plus interest, and the borrower defaulted after paying half of the loan, the co-signer would be responsible for paying the remaining half of the loan. SOURCE: FI:063 SOURCE: Littrell, J.J., Lorenz, J.H., & Smith, H.T. (2012). School to career (9th ed.) [pp. 524-528]. Tinley Park, IL: Goodheart-Willcox.

"What federal law must businesses follow when they offer revolving charge accounts and send customers updated statements showing the status of their accounts? A. Fair Credit Billing Act B. Equal Credit Opportunity Act C. Truth-in-Lending Act D. Fair and Accurate Credit Transactions Act"

C Truth-in-Lending Act. The Truth-in-Lending Act requires businesses to provide credit customers with all the information that applies to their accounts and send customers regularly updated statements showing the status of their accounts. The purpose of the Equal Credit Opportunity Act is to make the granting of credit fair by prohibiting the denial of credit based on the applicant's gender, race, age, marital status, or national origin. The Fair Credit Billing Act requires businesses that extend credit to respond within 30 days to any customer's complaint or inquiry about a billing error. The Fair and Accurate Credit Transactions Act gives consumers the right to inspect the files of their credit history at any credit agency and to have any mistakes corrected. SOURCE: FI:002 SOURCE: LAP-FI-002—Give Credit Where Credit Is Due (Credit and Its Importance)

"Aisha wants to buy new ovens for her bakery. They'll be expensive, but she has some money saved up that she can use for the purchase. Aisha plans to finance her new equipment using A. accounts receivable. B. dividends. C. debt. D. equity./"

D Equity. Assets a company already owns and can use to finance a new venture are known as equity. Aisha's using equity when she purchases new ovens with money she already has saved up. Dividends are sums of money paid to investors or stockholders as earnings on investments. Debt refers to taking out a loan from a bank or other lending institution. Since Aisha already has her money, she doesn't have to go into debt to buy the new ovens. Accounts receivable refers to all the money owed to the business by others. SOURCE: FI:354 SOURCE: LAP-FI-007—Money Matters (Role of Finance)

"When preparing her company's budget, Tatiana's manager told her that the company would be merging with another company. To maintain her ethics, Tatiana should A. report this information to the SEC. B. make sure the merger benefits the public interest. C. maintain her independence. D. keep this information confidential."

D Keep this information confidential. Confidentiality is the practice of keeping information secret or private. Accountants have a legal responsibility to protect their clients' information. Tatiana should therefore keep information about her company's upcoming merger confidential. Tatiana does not necessarily have any control over whether or not the merger benefits the public interest. There is no indication that Tatiana has a conflict of interest that would require her to maintain her independence. Also, there is no indication that Tatiana should report the information about the merger to the SEC. SOURCE: FI:351 SOURCE: LAP-FI-351—With Due Care (The Role of Ethics in Accounting)

"Incorrect information on an individual's credit report may hinder his/her ability to A. earn interest on stock holdings. B. withdraw money from a savings account. C. pay monthly credit card balances. D. obtain a bank loan."

D Obtain a bank loan. Banks obtain credit reports to make decisions about providing loans; therefore, it is important for individuals to periodically review the accuracy of their credit reports. By obtaining copies of their credit reports from an authorized credit agency, individuals can identify problems and take corrective action. Information presented in a credit report does not affect an individual's ability to withdraw money from a savings account, pay monthly credit card balances, or earn interest on stock holdings. SOURCE: FI:072 SOURCE: Irby, L. (2018, May 1). What is a credit report and why is it important? Retrieved September 19, 2018, from https://www.thebalance.com/what-is-a-credit-report-and-why-is-it-important960448

"Nico knows he would never do anything unethical, but he is sure that all of his coworkers are tempted at one point or another. Nico is demonstrating A. loss aversion. B. short-term gratification. C. framing. D. over-optimism and overconfidence."

D Over-optimism and overconfidence. People are often guilty of being overly sure of themselves, particularly in their ability to act ethically. In this case, Nico is overly confident in his own ability to resist unethical behavior, even though he recognizes that everyone around him faces temptations. Short-term gratification is the pursuit of instant satisfaction rather than delaying rewards for greater long-term benefit. Framing is the tendency to respond to situations based on how those situations are posed or viewed. Loss aversion is the tendency to avoid a loss at all costs, even when attempting to avoid that loss can lead to consequences. SOURCE: FI:355 SOURCE: LAP-FI-355—Money Morals (The Role of Ethics in Finance)


Set pelajaran terkait

Chapter 4: Advanced database systems

View Set

Emergency Preparedness Response Course (EPRC) - Clinician 8 hr

View Set

Sexually Transmitted Infections and Contraception

View Set

Describe: Classic Study: Loftus and Palmer's ROAD

View Set

oop with inheritance, polymorphisms ,encapsulation, and abstraction

View Set